Combined Book Medicine 2005 - 2019 (1998 AL - 2012 AL) MCQ
Combined Book Medicine 2005 - 2019 (1998 AL - 2012 AL) MCQ
Combined Book Medicine 2005 - 2019 (1998 AL - 2012 AL) MCQ
MEDICINE
1998 A/L – 2012 A/L
FACULTY OF MEDICINE
UNIVERSITY OF COLOMBO
PROJECT BY,
MFSU/MSWS
MFSU/MSWS 0
CONTENT
Page No.
1. Final MBBS Common MCQ Paper (1998 A/L) 2
2. Final MBBS Common MCQ Paper (1999 A/L) 7
3. Final MBBS Common MCQ Paper (2000 A/L) 15
4. Final MBBS Common MCQ Paper (2001 A/L) 19
5. Final MBBS Common MCQ Paper (2002 A/L)
6. Final MBBS Common MCQ Paper (2003 A/L) 23
7. Final MBBS Common MCQ Paper (2004 A/L) 25
8. Final MBBS Common MCQ Paper (2005 A/L)
9. Final MBBS Common MCQ Paper (2006 A/L)
10. Final MBBS Common MCQ Paper (2007 A/L) 32
11. Final MBBS Common MCQ Paper (2008 A/L) 38
12. Final MBBS Common MCQ Paper (2009 A/L) 42
13. Final MBBS Common MCQ Paper (2010 A/L) 48
14. Final MBBS Common MCQ Paper (2011 A/L) 53
15. Final MBBS Common MCQ Paper (2012 A/L) 58
MFSU/MSWS 1
FACULTY OF MEDICINE
UNIVERSITY OF KARAPITIYA
FINAL MBBS EXAM – 2005
1998 A/L BATCH
MEDICINE
MFSU/MSWS 2
d) Absent tendon reflexes in lower limb b) “helitrope rash”is universal
e) Marked sensory impairment c) ovarian carcinoma is etiologically related
13) Hepatic encephalopathy is characterized by d) muscle biopsy is contra indicated
a) Agitation e) complement deficiency is a feature
b) Exaggerated reflexes 21) Recognized neurological complication of
c) Acidotic breathing rheumatoid arthritis
d) Dilated pupils a) carpal tunnel syndrome
f) Fits b) optic neuritis
14) Gastro oesophageal reflux c) monouritis multiplex
a) can lead to oesophageala stricture d) spastic quadriceps
b) cannot be controlled by medication alone e) cerebellar ataxia
c) is common in pregnancy 22. Causes of macrocytosis in peripheries
d) Presents with Fe deficiency anemia a) haemolytic anaemia
e) is treated with eradication of H.pylori b) hyperthyroidism
15) Nephrotic syndrome occurs due to c) alcoholic liver disease
a) focal segmental glomerulonephritis d) strict vegetarian
b) SLE e) methotrexate
c) Amyloidosis 23) Regarding chronic granulocytic leukemia
d) Diabetic Mellitus a) rare in below 20 years
e) Penicillamin therapy b) present with dragging sensation in left
16) Acute pancreatitis occurs due to hypochondriac sensation
a) mumos c) to confirm the diagnosis bone marrow
b) vivax malaria examination is essential
c) alcohol d) with the treatment of Philadelphia
d) paracetamol chromosome level reduces
e) hypertriglyceridemia e) early treatment is not necessary
17) WOTF of renal calculi 24) Thrombocytopenia occur
a) commoner in females’ a) long term treatment with steroids
b) hypercalciuria is the most common b) splenectomy after accidental rupture
metabolic abnormalities c) IMN
c) medullary sponge kidney is a predisposing d) CML
condition e) mycoplasma infection
d) pure uric acid stones are radio lucent 25) Features of panhypopitiurism
e) stones greater than 10mm in diameter a) Amenorrhoea
usually requie intervention b) increased fasting blood sugar
18) Acute renal failure is recognized complication of c) Reduced TSH and T level
a) Cholera d) reduced libido
b) SLE 26) Thyrotoxicosis
c) Organophosphate poisoning a) can present without palpable nodule
d) Rhabdomyolitis clinically
e) Falciparum malaria b) can be excluded when weight gain is present
19) Ankylosing sondilytis c) can be present with exophthalmos when the
a) Mainly affect old females patient is having graves disease
b) usually painless d) propanalol is useful drug during treatment
c) limits chest expansion e) can be treated with carbamezapin and
d) accompanied by aortic stenosis thyroxin combination
e) common in sacroiliac joints 27) WOTF
20) Dermatomyositis a) secondary amenorrhoea seen in turner’s
a) muscle weakness is universal syndrome
MFSU/MSWS 3
b) oligospermia means absence of sperms d) Neurotoxicity is a common feature of
inejaculatory fluid Russell viper bite
c) virilisation means female secondary sexual e) Rattle snake is a deadly poisonous snake
characteristics occur in males found in Sri Lanka
d) impotence is the inability of male to achieve 34) Features of alcohol withdrawal include
or sustain an erection adequate for satisfactory a) flapping tremor
intercourse b) hypotension
e) libido refers to sexual desire or interest c) bradycardia
28) Notifiable diseases in Sri Lanka d) visual hallucination
a) malaria e) seizures
b) community acquired pneumonia 35) T/F
c) food poisoning a) Triceps muscle is supplied by C8,T1 root
d) cystitis due to Klebsiella b) shoulder abduction done by C5,C6 roots
e) leptospirosis c) C7 dermatome supplies middle finger
29) Which of the following infections present with d) hip flexion done by L1 root
rash e) plantar flexion done by S1 root
a) rubella 36) Statins
b) malaria a) It induces HMG Co A reducta
c) scrub typhus b) Can be used as secondary prevention of
d) Herpes zoster hyperlipidemia in heart disease.
e) cholera c) contraindicated in Pregnancy
30) 30 yr old farmer presented with snake bite. d) are better than Gemfibrozil for control LDL
Following are the indications for anti venom therapy. cholesterol
a) clotting time > 20 min e) Myalgia is a known complication.
b) hypotension (37)Drugs used in typhoid fever
c) pain on die bitten limb a) Ciprofloxacin
d) bilateral partial ptosis b) Co-Teimoxazole
e) respiratory paralysis c) Cefotaxime
31) Hyperkalemia d) Co-amoxyclave
a) results in severe muscle weakness e) Pivmeccilline.
b) in a complication of rhabdomylosis 38) regarding z score
c) is manifested by U wave in ECG a) It examine the sample men with mode of
d) is reduced with salbutamol inhaler population.
e) readily responds to treatment with b) in normal population 95% of people lie in
salbutamol & dextrose 1.1 z score.
32) Features of pseudohypoparathyroidism c) It gives idea regarding the deviation of
a) subnormal level of parathyroid hormone particular value from the mean.
b) increase of ca2+ in urine d) Standard error is necessary for calculate z
c) has association with hypothyroid score.
d) all target tissues resistant to parathyroid e) It is simple statistical test.
hormone 39) Disease inherited in Autosomal dominant pattern.
e) short metatarsals a) Thalassemia.
33) T/F regarding snake bites in Sri Lanka b) Hereditary spherocytosis.
a) Ceylon krait commonly found in dry zone c) Hemophilia A
b) Saw scaled viper bites are reported in d) G6PD deffiency.
Anuradhapura district e) Vitamin FD resistant rickets.
c) Hump nosed viper bites carry a high 40) Lesions in psoriasis is,
mortality rate a) Erythematous
b) Itchy
MFSU/MSWS 4
C) Improve with sunlight.
d) Associated with sacroilitis.
e) Also found in mouth.
MFSU/MSWS 5
FACULTY OF MEDICINE
UNIVERSITY OF PERADENIYA
FINAL MBBS EXAM – 2005
1998 A/L BATCH
MEDICINE
8) Features of life threatening attack of bronchial
asthma,
1) In cardiac tamponade, a) Bradycardia
a) Associated with pulmonary oedema b) Pa CO2 < 40 mmHg
b) IV diuretic has a beneficial effect c) Pa O2 < 60 mmHg
c) Pulses paradoxus d) Arterial pH above 7.46
d) Increase JVP in inspiration e) Can say ten words in one breath
e) Hypothyroidism is a cause 9) Conditions leading to cerebral abscess
3) In heart failure, includes,
a) Anaemia is a cause of low output a) Aortic stenosis
cardiac failure b) Fallot's tetralogy
b) Peripheral vasoconstriction occurs c) Middle ear infections
c) ACE inhibitors reduce mortality d) Mitral stenosis
d) Renin Angiotensin system activated e) Viral meningitis
e) Implanted devices have a place 10) Features of Guillian Barre,
4) Pt presented with recurrent urinary calculi. a) Severe wasting of lower limb muscles
What are the investigations you would do? b) Extensor plantar response
a) Fasting serum Ca+2 c) Paraesthesia of lower limb
b) Urine culture d) Decrease conductive velocity of nerves
c) Renal biopsy e) Increase polymorpho nuclear cells in
d) Urinary excretion of amino acids the CSF
e) FBS 11) What are the causes of absence of tendon
5) WOF is/are T/F of pleural effusion, reflexes?
a) Lobar pneumonia is a cause a) Proximal myopathy
b) >3.5 g/dL suggests transudate b) Myesthenia Gravis
c) Lateral decubitis XR useful to detect c) Diabetic neuropathy
small PE d) Tabes dorsalis
d) Pleural biopsy done using Abram's e) Guillian Barre
needle 12) Causes of ptosis,
e) Due to TB, PE fluid characteristically a) Guillian Barre
has high AFB b) Hump nosed viper envenomation
6) WOF is/are T/F regarding type II respiratory c) Myesthenia Gravis
failure, d) Posterior communicating artery
a) Hypoxia is a feature aneurysm
b) Hypocarbia is a feature e) Horner's syndrome
c) Severe pneumonia is a recognised 13) Treatment options for secondaries in liver of
cause colorectal,
d) GBS is a recognised cause a) Surgical excision
e) Treated with 60% O2 is indicated b) Cryotherapy
7) Fibrosing alveolitis, c) External beam radio therapy
a) A disease of restrictive type d) Chemotherapy
b) Cause clubbing e) Lazer therapy
c) End respiratory crepitation 14) Fulminant hepatic failure,
d) Respond to steroid a) Commonly caused by aspirin poisoning
e) Usually cause productive cough b) Hep-D virus is a common cause
MFSU/MSWS 6
c) May not cause jaundice a) Could cause acute flaccid weakness of
d) Can lead to chronic liver disease lower limbs
e) Heoatomegaly is not a feature b) Could be exacerbated by alcohol
15) Possible factors causing hepatic c) Phenobarbitone could be used safely
encephalopathy, d) Could be treated with steroids
a) Varicial bleeding e) Could present as a surgical emergency
b) Low protein diet 32) Following arrhythmias can occur due to
c) Spontaneous bacterial peritonitis ingestion of 6 seeds of yellow oleander,
d) Thiamine therapy a) 1st degree heart block
e) Constipation b) Bundle branch block
16) Retardation of progression of CRF in diabetic c) Complete heart block
nephropathy can be achieved by, d) Atrial tachycardia
a) Maintaining < 130/80 mmHg e) Ventricular tachycardia
b) Controlling of blood sugar level 33) WOF is/are T/F regarding kidney,
c) Erythropoetin therapy a) It is a intra peritoneal organ
d) Treatment of hypocalcaemia b) Develop only from mesonephrone
e) Treatment of UTI 35) Which of the following is/are true regarding
17) WOF disease/s due to immunological damage, the kidney,
a) Diabetic nephropathy a) It is a intra peritoneal organ.
b) Lupus nephritis b) Develop only from mesonephrone
c) Good-Pasture's syndrome c) Left kidney hilum is higher than right
d) Renal TB d) Ureters lay alone the transverse processes
e) Analgesic nephropathy e) Kidney move with respiration
18) In a patient with polyarthritis, the diagnosis is 36) WOF is/are T/F regarding drugs used in OP
more likely to be rheumatoid arthritis than poisoning;
osteoarthritis in the presence of, a) IV atropine is the drug of choice
a) Elevated ESR b) Pralidoxime can reactivate the choline
b) Symmetrical involvement of proximal esterates
interphalangeal joint c) Pralidoxime should be given early
c) Involvement of tempero-mandibular d) Can cause acute renal failure early
joint e) Papillary constriction indicate adequate
d) Presence of rheumatoid factor atropinisation
e) Symptomatic response to steroid 37) Epidemiological factors in srilanka;
19) Macrocytic anaemia can be seen, a) Commonest hospital deaths are due to
a) Myalodysplastic syndrome diarrheal diseases
b) Thalassemia trait b) Commonest OPD presentation is respiratory
c) Renal failure disorders
d) Phenytoin therapy c) Prevalence of DM is 0.2%
e) Rheumatoid arthritis d) Type 1 Diabetes is commonest
23) Hereditary cause of DM suggests, e) Malaria epidemic occurred at 2004
a) HLA-3 38) AS
b) Insulin molecular change a) Common after 40y
b) Male: Female is 9:1
c) Increase prevalence of hi Europe c) For prognosis shoulder test
d) Similar to type 1 DM d) Aortic regurgitation associated
26) X linked inheritance, e) Multi systemic
a) Haemophilia
b) Vit D resistant rickets
c) Neurofibromatosis
d) Downs syndrome
e) Haemochromatosis
30) Porphyrias,
MFSU/MSWS 7
FACULTY OF MEDICINE
UNIVERSITY OF RAGAMA
FINAL MBBS EXAM – 2006
1999 A/L BATCH
MEDICINE
08). T/F regarding pneumonia
a) Common cause of community acquired
02) Wide fix splitting 2nd heart sound pneumonia caused by mycoplasma pneumonia
a) Uncomplicated ASD b) Smoking is a risk factor
b) Tetralogy of fallot c) Positive blood culture indicates a poor
c) Aortic stenosis prognosis
d) Right bundle branch block d) Hyponatreamia may occur
e) Constrictive pericarditis e) Hospital acquired pneumonia mostly due to
03) Exercise stress testing gram (+) ve bacilli
a) Usually done by treadmill 09) Recognize causes for seizures
b) May give false (+) ve in young people a) Guillen barre’s syndrome
c) Provide prognostic information b) Hypoglycemia
d) May cause disease in BP in sever coronary c) Hypocalcaemia
arterial disease d) Motor neuron disease
e) Is said to be (+) ve if ST segment depressed e) Chronic renal failure
with unslopping ST segment 11 A 35 year old male presented with a sudden onset
04) Bradicardia of sever occipital headache. T/F regarding following
a) Seen in all athletics – vagal tone dela a) Optic fundus examination may help in
b) Follows the treatment of Ca+2 channel diagnosis
blockers b) Absent neck reflex within 2 hours will
c) Seen in inferior MI exclude a sub arachnoid hemorrhage
d) Associated feature of sick sinus syndrome c) Purpuric patches may give a clue to the
05) In bronchial asthma diagnosis
a) Inflammation of the air way is a feature d) Migraine may present with similar clinical
b) Aspirin causes exacerbation features
c) Prophylactic use of steroids is indicated in e) CT scan will show abnormalities if it is due
intermittent asthma to sub arachnoid haemorrhage
d) Silent chest is a marker of severity
e) Spacer device use to increase the drug
delivery to airway
06) Regarding bronchiectasis T/F
a) Cyanosis
b) Clubbing
c) Hyperinflated chest
d) Halitosis
e) Bibasal end inspiratory fine crepts
07) Normal telechest
a) Right hilum is higher than left hilum
b) In deep inspiration to 5th 6th ribs
c) Trachea deviated to right side at aortic
knuckle
d) Medial end of the clavicle same distance
from the sinus process
e) Right heart border is made by the right
ventricle
MFSU/MSWS 8
FACULTY OF MEDICINE
UNIVERSITY OF JAFFNA
FINAL MBBS EXAM – MAY 2006
1999 A/L BATCH
MEDICINE
c) Tachycardia
01) Chronic heart failure d) Acidosis
a) Reduced ejection fraction is essential feature e) Well respond to IV frusemide
b) Beta blockers contraindicated in systolic 8) Features of COPD
dysfunction a) Normal functional residual volume
c) ACEI is a 1st line treatment in systolic heart b) Reduced FEV1
failure c) Normal FEV1/FEV ratio
d) Digoxin is only indicated if AF is developed d) Low transfer factor (DLCO)
e) Natriuretic peptide is reduced e) Total lung volume reduced
02) Which of the following drugs prolong survival in 9) Causes of peripheral neuropathy
MI a) Therapy with INAH
a) Streptokinase b) Paraquat poisoning
b) Aspirin c) Vit B12 deficiency
c) Nitrates d) DM
d) Morphine e) Leprosy
e) Hydrocortisone 10) 25 years old female presented with bilateral ptosis
03) Rheumatic fever worsening in evening and progressive increasing SOB.
a) Causes erosive arthritis Which of the following T/F
b) Commonly seen in children than I adults a) Exaggerated tendon reflexes
c) Chorea occurs as a late feature b) Bilateral constricted pupil
d) Commonest cause for pure aortic stenosis in c) Papilloedema
srilanka d) Lung function testing show obstructive type
e) Cannot be diagnosed when normal ASOT e) CXR reveal anterior mediastinal mass
04) Uncomplicated osteum secondum ASD 11) in a patient with increased intracranial pressure
a) Fixed split of 2nd heart sound (ICP)
b) Central cyanosis a) Headache is worse on coughing
c) ESM in pulmonary area b) Cystolic pressure usually falls
d) RBBB in ECG c) Papilloedema is present in over 90%
e) High risk of infective endocarditis d) LP is urgently indicated
5) Bronchiectasis e) Presence of 6 th nerve palsy localizes the
a) Common in lower lobe than in upper lobe lesion to the brain stem
b) Hemoptysis is known to occur 12) Ina patient with paraplegia spinal cord disease
c) Measles in childhood is an etiological factor suggested by
d) Mycoplasma pneumonia is the common a) extensor plantar response
cause for infective exacerbation b) sensory level
e) Postural drainage is an effective therapy c) loss of touch sensation below knee
6) Feature of acute severe asthma d) positive straight leg raising test
a) Tachycardia PR > 110/min e) atonic bladder
b) Pulsus bisferians 13) Clinical and laboratory findings of viral hepatitis
c) Impaired percussion note in both lung field include
d) PEFR of 600L/min a) tender hepatomegaly
e) Unable to complete one sentence in a breath b) subconjunctival haemorrhage
7) Characteristic feature of ARDS c) lymphadenopathy
a) Hypoxia d) urine urobilinogen increased in recovery
b) Hypercapnea phase
MFSU/MSWS 9
e) lymphocytosis d) Rheumatoid arthritis- ASOT increased
14) Chronic active hepatitis aetiology associated with e) Gullain Barre syndrome- increase protein
a) Hepatitis E with decreased cells in CSF
b) Alcohol 22) β thalassemia intermedia
c) Wilson’s disease a) chronic leg ulcer is a feature
d) SLE b) spleen is not enlarged
e) Methyldopa c) Hb % is usually bebv/4g/dL
15) Following disorders are recognized sequale of d) Hb A2 increased
chronic alcoholism e) Iron overload is a complication
a) Atrial fibrillation 23) Following measures are best avoided in these
b) Nephrotic syndrome conditions
c) Dilated cardiomyopathy a) IV injection- bleeding disease
d) Chronic pancreatitis b) PR examination- severe leucopenia
e) Dementia c) LP in transverse myelitis
16) Post streptococcal glomerulonephritis d) Draining- osteoarthritis joint
a) Group B streptococci is a causative factor e) Liver biopsy- low platelet count
b) Often followed by infection of throat than 24) T/F regarding haemophilia A
skin a) Severity of disease same among all affected
c) Nephrotic range of proteinuria known to family members
occur in initial period b) X linked dominant condition
d) Often leads to CRF c) Cryo poor plasma is effective in Rx
e) HCT given to treatment for oedema d) Severity reduced with advancing age
17) Which of the following give clue to cause of CRF e) Plasma derived factor VIII concentration
a) Oedema transfusion carries risk transmission of malaria
b) Hypertension 25) 50 years old man DM, severe pneumonia,drowsy,
c) History of DM BP~70/30, PaO 2 50mmHg, pH ~ 7.2, Blood sugar ~
d) Normochromic anemia 370mg/dL
e) Features of prostatic hypertrophy a) Water resuscitation with saline
18) Acute nephritic syndrome is a recognized b) IV dobutamine to increase BP done initially
complication of to
a) SLE c) High blood sugar should be reduced to 90
b) HSP mg/dL
c) DM d) Intubation and ventilation indicated
d) Group B streptococci e) Low dose hydrocortizozne is useful
e) Falciparum malaria 26) Mx of diabetic kedoacidosis
f) Amyloidosis a) Insulin is best given by continuos infusion
19) SLE b) Bicarbonate administration is indicated in
a) Disease of adolescent any severity of acidosis
b) Treated with methotrexate c) Thrombo prophylaxis is contra indicated
c) Cause avascular necrosis of hip d) K+ is given only if serum K is low
d) Cause endocarditis e) Hypotonic saline indicate if Na+ above
e) Caused by penicillamine 150mmol/L
20) T/F regarding correct combination 27) Features of acromegaly
a) DIPJ- rheumatoid arthritis a) Osteoporosis
b) Sacro iliac joint- SLE b) Increased renal mass
c) PIPJ- primary osteoarthritis c) Increase depth of frontal sinus
d) 1 st metartarsal joint- gout d) Thickening of peripheral nerves
e) Upper cervical joint- cervical spondylosis e) Macroglossia
21) T/F regarding correct combination 28) Regarding infectious mononudeosis (IMN)
a) in SLE- normal CRP a) Caused by retroviral infection
b) Osteoporosis- inceased ALP b) Cause massive hepatomegaly commonly
c) Multiple myeloma- decreased Ca 2+
MFSU/MSWS 10
c) Painful cervical lymphadenopathy excludes e) Severe muscle weakness
the diagnosis 36) Which of the drugs correct in treatment of
d) Autoimmune hemolytic anemia is a known following conditions
complication a) Intestinal amoebiasis- metronidazole
e) Burkitt’s lymphoma is a recognized sequale b) Thyrotoxicosis- carbamzipine
29) Rx recommended for complicated falciparum c) SVT- lignocaine
malaria d) Mycoplasma pneumonia- cephalexin
a) Chloroquine e) Psoriatic arthropathy- chloroquine
b) Fansidar 37) In a patient with respiratory tract infection TB
c) Quinine treatment with
d) Mefloquine a) Erythromycin
e) Artemethor b) Clarithromycin
30) WOTF disease spread by mosquito c) Coamoxi-clav
a) Dengue d)Ciprofloxacin
b) Leishmaniasis e) Pivambicilin
c) Yellow fever 38) In a study of measuring blood pressure among
d) Typhus 1000 children the result mean is 100 mmHg &SD
e) Plaque is 5, following are T/F
31) Isotonic saline a) Blood pressure measurements show normal
a) Used as a vehicle in infusing sptreptokinase distribution
b) Fluid of choice in a patient with diabetic b) Most of the children have BP between
ketoacidosis with normal blood pressure 90/110 mmHg
c) Has added K + c) Half of the children have BP below than 100
d) Should be given cautiously for the fear of mmHg
hypersensitivity reaction d) Median is 100 mmHg
e) The sodium level was low compared to e) 60 year old man has more BP than children
plasma of same volume 39) Following are correct combinations
32) T/F regarding hyperthyroidism a) Neurofibromatosis- autosomal dominant
a) Serum phosphate level elevated b) Haemochromatosis- Autosomal recessive
b) Renal excretion of phosphate reduced c) G-6PD deficiency- autosomal dominant
c) Serum calcium level elevated in response to d) Marfan’s syndrome- autosomal dominant
thiazide diuretic treatment e) Colour blindness- X linked recessive
d) Usually detected clinically 40) Cigarette smoking
e) In young patient surgery indicated only if a) Predispose to cirrhosis
symptoms persists b) Associated with CA bladder
33) Polyneuropathy occurs c) HDL increase
a) Organophosphate poisoning d) Cause less harm to women compared to
b) Arsenic poisoning men
c) Lead poisoning e) Associated with malignant HT
d) Gloriosa superba poisoning
e) Russell’s viper bite
34) Features associated with opioid poisoning
a) Large pupil
b) Hypotension
c) Respiratory depression
d) Tachycardia
e) Muscle fasciculation
35) UMN lesion differs from LMN lesion by having
a) Reduced reflex
b) Hypertonia
c) Fibrillating potential in EMG
d) Fasciculation
MFSU/MSWS 11
FACULTY OF MEDICINE
UNIVERSITY OF COLOMBO
FINAL MBBS EXAM (MAIN) –2006
1999 A/L BATCH
MEDICINE
a) Organophosphate
b) TCA
Part II (Multiple Choice Questions) c) Kaneru
a) Volume difference of radial pulses in d) Verapamil
both arms e) Cannabis
b) Bruit over the abdomen above and 14) 56 years old obese, DM patient develops
lateral to the umbilicus sudden pain in left elbow, other joints are not
c) S. Cholesterol 350mg/dL swollen or painful. Possible causes,
2) A 55yrs man presented with acute chest pain. a) Olecranon bursitis
Which of the following can differentiate acute b) Septic arthritis
from unstable angina, c) Reiter's arthritis
a) Profuse sweating d) Gout
3) T/F, 15) C/l in pregnancy,
a) Kerly B lines- consolidations a) Acyclovir
b) Isoniaside inhibit hepatic drug b) Captopril
metabolism c) Carbimazole
4) Following are associated, d) Atorvastatin
a) Tinnitus - Menier's disease 16) An 18 years old child has nephrotic syndrome.
b) Numbness of one side of face - lateral Which investigations will help in the diagnosis
medullary syndrome of minimal change glomerular nephritis,
c) A normal CSF report a) Measurement of serum cholesterol
d) Seizures b) Serum protein electrophoresis
e) Splenomegaly c) Urine protein electrophoresis
8) Features that require endoscopy in a patient d) Examination of centrifuged deposits of
with dyspepsia, a urine sample
a) Recurrence of symptoms after e) Measurement of serum creatinine
omeprazole therapy
9) HIV, a) Carbimazole,
a) Known to infect CD4 cells 17) True/False,
10) Vertical nystagmus, a) A bolus dose of quinine is
a) Finger nose incoordination recommended in patients with
b) Cerebellar-Pontine angle tumours complicated falciparum malaria
c) Vestibular neuritis b) Primaquine elemenates the liver
11) In a patient with fever and unconsciousness hypnozoites in P. vivax
which factors are in favour of viral encephalitis Proguanil is used in malaria prophylaxis
than cerebral malaria,
a) Neck stiffness
b) Abnormal CSF analysis
c) Drowsiness
d) Seizures
12) 30 years old female, Hb 6 g/dL, features
suggesting haemolytic anaemia rather than
aplastic anaemia,
a) WBC - 1500
13) Tachycardia by overdose,
MFSU/MSWS 12
FACULTY OF MEDICINE
UNIVERSITY OF COLOMBO
FINAL MBBS EXAM –2006
1999 A/L BATCH
MEDICINE
6) Liver biopsy,
1) Regarding cardiomyopathy, a) Most commonly done through
a) Dilated type seen in alcoholics intercostal route
b) Sudden death occurs in hypertrophic b) Bleeding likely within 3 hours
cardiomyopathy c) Sedation is important
c) Ventricular filling is impaired in d) Easy to do when there is ascites
restrictive type e) U/S hepatobilliary system should done
d) Mirral stenosis murmur heard in before
HOCM 7) U/S examination shows enlargement of
e) p blockers contra-indicated in HOCM kidneys in,
2) Bad prognostic sign of MI, a) Acute renal failure due to pre renal
a) Female sex cause
b) Inferior Mi b) Amyloidosis
c) Ventricular fibrillation after c) Chrnoic glomerulonephritis
streptokinase treatment d) Diabetic nephropathy
d) Normal BP e) Acromegaly
e) Age 40 8) Prolenuria if more than 3.5g in 24 hours could
3) DM nephropathy, occur in,
a) One of the earliest changes is
thickening of GBM a) SLE
b) Smoking is a risk factor b) DM
c) HT & micro albuminuria is not c) Cardiac failure
compatible finding d) Tubular proteinuria
d) Angiotensin II receptor blockers are not e) Hypertension
useful in management 9) In a patient complaining indigestion which of
e) Metformin is a better drug to control the following indicates severe GIT problem,
diabetes with established DM a) Weight gain
nephropathy b) Anaemia
4) Prolactin levels increased in, c) Malaena
a) Pregnancy d) Lymphadenopathy
b) Primary hypothyroidism e) (+) test for h. pylori
c) Bromocriptine therapy 10) Step IV management of BA,
d) Polycystic kidney disease a) High dose inhaled steroids
e) Chiorpromazine therapy b) Regular long acting bronchodilator
5) c) Trial of leukotriene receptor blocker
a) Aspiration is better done with the d) Oral corticosteroid
patient in siipine'pbsiti, e) IV antibiotics
b) Biopsy is done after aspiration of the 11) In a patient with radial nerve pulse,
fluid a) Extension of shoulder affected
c) If pleural effusion due to TB, biopsy is b) Extension of elbow affected
usually negetive for c) Extensive sensory loss occur
d) Rheumatoid arthritis the pleural fluid is d) Extension of metacarpo phalangeal
sugar less joint
e) Empyema better aspirated with 16 G e) Sensory loss over the digits
needle
MFSU/MSWS 13
12) Restlessness and hyperactivity caused by e) Depression and physical illness may
following, present coincidentally
a) Hypomania 19) Possible sequalae to a traumatic event include,
b) Agitated depression a) Shock
c) Drug induced state b) Guilty feeling
d) Schizophrenia c) Grief reaction
e) Post ictal confusion d) Exacerbation of preexisting psychiatric
13) T/F regarding nerve lesions, condition
a) Distal ulnar lesion cause more e) Re experiencing
disability than proximal
b) Common peroneal nerve - weak plantar
flexion
c) Complete 3rd nerve lesion commonly
present with diplopia
d) 6th nerve lesion cause divergent squint
e) Median nerve lesion- weak pincer grip
14) Following are features of scrub typhus,
a) Average IP is 10 days
b) Appearance of a painless eschar
c) Presentation without fever
d) Generalised pustular rash
e) Formation of a membrane on the
tonsils
15) Obesity seen in,
a) Prader-Willi syndrome
b) Hypothalamic disease
c) Addison's disease
d) Treatment with phenothiazine
e) Primary hypothyroidism
16) Good indicator for Schizophrenia,
a) Acute onset
b) Positive family history
c) High expressed emotions in the family
d) Presentation with prominent affective
symptom
e) Good response to treatment
17) Following are recognised symptoms of
anxiety,
a) Apprehension
b) Inattention
c) Fear of impeding danger
d) Peaceful mind
e) Persecutary delusion
18) Correct correlation regarding depression and
physical illness,
a) Depression leads to physical illness
b) Depression affects prognosis of
physical illness
c) Physical illness may cause depression
d) Treatment of some physical illness
cause depression
MFSU/MSWS 14
FACULTY OF MEDICINE
UNIVERSITY OF JAYAWARDENEPURA
FINAL MBBS EXAM – 2006 (MAIN)
1999 A/L BATCH
MEDICINE
d) Reduced iron stores
e) Improve with oral iron
1) Regarding stable angina 9) Causes of Reynolds phenomenon include
a) Resting ECG normal up to 50% a) Rheumatoid arthritis
2) Side effects of carbamazepine include b) Scleroderma
a) Diplopia c) Rheumatic fever
b) Ataxia d) Cervical rib
c) Gum hyperplasia e) Carpal tunnel syndrome
3) 20 yrs. old girl admitted with a history of 10) Rx of type II DM
ingestion of 30 pcm 1 hour ago. Strategies in a) Glibenclamide is the drug of choice
management are in elderly
a) Immediate estimation of blood pcm b) Metformin reduce the appetite
level c) Tolbutamide better than
b) Immediate administration of acetyl glibenclamide in patients with renal
cysteine failure
c) Gastric lavarge 11) Association of hypothyroidism
d) Activated charcoal a) Type II DM
e) Furosemide b) Increase CPK
4) Recognized causes of nephrotic syndrome c) Carpal tunnel syndrome
include d) Hypercholesterolemia
a) Malaria e) Nerve deafness
b) Focal glomerulosclerosis 12) Cause of precocious puberty in male child
c) IgA nephropathy include
d) SLE a) Congenital adrenal hyperplasia
5) Regarding UFR b) Testicular tumours
a) Few granular cast not significant 13) Falciparum malaria
b) Few pus cells do not exclude UTI a) Lifelong immunity acquired after an
c) RBC cast indicate glomerular acute attack
damage b) Acute renal failure is a complication
6) Manifestation of multiple myeloma include c) All patients treated with quinine
a) Acute paraplegia d) Chloroquine in effective in
b) Acute renal failure prophylaxis in SL
c) Acute confusional state e) Primaquine for 14 days reduces the
d) Retinal hemorrhage relapse
e) Maculopapular rash 14) Parient with bronchial carcinoma presented
7) 40yrs old female presented with tiredness Ix- with Na+ 110 meq/l & K+ 4.5 meq/l.
Hb 9g/dl, WBC 2.5 x 109 , platelet 100 x 109 Which of the following investigations should
a) Acute leukemia be done to diagnose metabolic acidosis?
b) ITP a) Urine osmolality
c) SLE b) Serum osmolality
d) Aplastic Anaemia c) ABG
e) G6PD deficiency d) Urine 24 hour K+ level
8) Anaemia in chronic diseases e) Urine 24 hour VMA level
a) Mild disease
b) Macrocytes present
c) Fe absorption poor
MFSU/MSWS 15
FACULTY OF MEDICINE
UNIVERSITY OF JAYAWARDENEPURA
FINAL MBBS EXAM – DECEMBER 2006
2000 A/L BATCH
MEDICINE
d) Reduced RBC life span
e) BM suppression
1. T/F regarding the therapeutic measures of a patient 17. Regarding nephrotic syndrome
with shock a) Painless recurrent hematuria is a feature
a) Adesonian crisis – methyl prednisolone b) Resembles histology of diabetic nephropathy
b) Complete heart block –cardiac packing c) Relapse following an URTI
c) Anaphylactic shock – IM Adrenalin 18Haematuria
2. T/F regarding atrial fibrillation a) Loin pain needs immediate IVU
a) Digoxin reverts rhythm to sinus rhythm b) Terminal haematuria indicate pathology in
5. Treatment of tuberculosis bladder
a) Patient should be isolated until sputum AFB c) Increased acid phosphatase confirm the
becomes negative diagnosis prostate CA
b) 4 drug regime is given for 4 months d) Dysmorphic RBC indicate glomerular
c) INAH is given for contacts damage
d) 9 month drug regime is the one used
generally 20. Regarding treatment of rheumatoid arthritis
10. Which of the combinations are correct a) Negative rheumatoid factor excludes the
a) Sensory level at D10 – transverse myelitis diagnosis p
b) Spastic weakness of the lower limbs - spinal b) Ibuprofen has disease modifying properties
cord compression c) Lefluenemide is used
11. 60 yrs. old male c/o vertigo, nystagmus, R/s finger d) DMARDs are started as soon as possible
nose test affected, sensory loss on R/s of face & L/s of after diagnosis
body. Where are the possible sites of the lesion? 24. T/F regarding Multiple myeloma
1) CP angle tumour a) Causes pathological fractures
2) R/S pons 26. Clinical biochemical features of primary
3) R/ Cerebellar hemisphere hypothyroidism include
4) L/Parietal lobe a) Increase TSH
5) MLE b) Reduced serum cholesterol levels
12. T/F regarding the management of thrombotic c) Increase nerve conduction velocity
stroke d) Reduced MCV
a) Recombinant t-PA e) Increased cardiovascular ratio
b) Carotid end arterectomy 27. Acute management of DKA
c) Aspirin a) S/C insulin in used
d) IV Mannitol 28. T/F regarding following
e) IV hydrocortisone a) H5N virus is the cause for the reent avian flu
13. Dysphagia seen in epidemic
a) Aortic aneurism b) Aciclovir inhibits the viral nucleic acid
b) Systemic Sclerosis synthesis
c) Myasthenia gravis c) Herpes zoster causes genital herpes
15. Omeprazole 29. Characteristic features of typhoid,
a) Is a H2 receptor blocker a) Thrombocytosis
16. Causes of Anaemia due to chronic renal failure b) Lymphadenophathy
a) Iron deficiency due to loss of appetite c) splenomegaly
b) Reduced erythropoietin levels d) Dysuria & frequency
c) GI bleeding
MFSU/MSWS 16
e) Rose spots
31. Which of the conditions will increase serum Ca2+
levels
a) Multiple myeloma
b) Secondary hyperparathyroidism
c) CRF
d) Pott’s disease
e) Bronchial carcinoma
32. Osteoporosis is a complication of
a) Heperthyroidism
33. Amitryptallin overdose includes
a) Dilated pupils
b) SVT
c) Peripheral neuropathy
d) Impaired liver function
e) Urine retention
36. Impotence is an adverse effect of
a) Methyldopa
38. Group of adults of 20 – 40 yrs. has serum
cholesterol level in a normal distribution. Mean is 190
mg/dl. SD = 20 mg/dl
a) 95% population will have cholesterol within
170-210 mg/dl
b) 50% population has cholesterol > 190 mg/dl
c) 95% population has cholesterol 150 mg/dl
d) In most of adults serum cholesterol is
190mg/dl
e) An independent 2 sample t – test cab be used
to compare the serum cholesterol levels
between males & females
40. Clubbing is a feature of
a) Bronchial asthma
b) Infective endocarditis
c) Bronchiectasis
d) Chronic liver failure
e) CRF
MFSU/MSWS 17
FACULTY OF MEDICINE
UNIVERSITY OF COLOMBO
FINAL MBBS EXAM –2007
2000 A/L BATCH
MEDICINE
d) Milliary mottling is characteristic in X-
1) A patient with rheumatoid arthritis attends a ray
medical clinic complaining of progressive e) Postural drainage is helpful
shortness of breath. WOTF features favour a 8) 19 years old female weighing 40kg admitted 8
diagnosis of fibrosing alveolitis? hours after ingesting 30 tablets of
paracetamol.
a) Finger clubbing
a) Perform gastric lavage immediately
b) Fine basal; crepitations
b) Activated charcoal
c) Chest radiograph showing bilateral c) Liver transaminase done before starting
apical cavitations antidote
d) Reductions of the lung volume d) Antidote contradicted if pregnant
e) Reduced FEV1/FVC ratio e) Liver failure regimen started on
2) Following therapeautic measures are T/F, admission
a) Paracetamol should be avoided in 9) Depressed farmer presented with self
chronic liver failure poisoning. Features suggestive of
b) Nifedipine useful in the management of organophosphate poisoning,
chronic heart failure a) GCS ⅗
3) What are the investigations useful in the b) Tachycardia
diagnosis of chronic diarrhoeal illness? c) Froth at mouth
d) Muscle twitching
a) X ray abdomen
e) Severe ulceration on tongue
b) Serum TSH
10) Regarding Dengue fever,
c) Stools occult blood a) Platelet rich plasma is routinely
d) Stool osmolality given if platelet count is less than
e) Blood urea level 50 000/ml
4) In the end stage renal failure 11) Rheumatoid arthritis,
a) Looser’s zones are visible in pubic a) Is a common cause for ITP
rami b) ANCA is positive
b) High phosphate levels c) Cause lumbosacral stiffening
c) Low calcium levels d) Causes sacroilitis
d) Low alkaline phosphatase levels 12) Neuromuscular junction dysfunction occurs
e) High uric acid levels in,
5) Glomerular nephritis a) Ceylon Krait bite
b) Guillian Barre syndrome
a) Reduced level of serum C3 in acute
c) Organophosphate poisoning
infective glomerulonephritis d) Myesthenia Gravis
6) Nerve conduction studies are useful in, 13) Following associates are true regarding seizure
a) Carpal tunnel syndrome disorders,
b) Myesthenia a) Petit mal seizures - 3 Hz wave and
c) Alzheimer's disease spike
d) Multiple Sclerosis b) Generalised unconsciousness
7) Bronchiectasis, c) Déja Vu temporal lobe epilepsy
a) Haemoptysis is a presenting feature d) Seizures in elderly - vascular lesion
b) Commonly present in lower lobe 14) Cushing syndrome,
c) Fine crackles characteristic a) Buffalo hump
MFSU/MSWS 18
b) Proximal myopathy
c) Striae
d) Hypokalaemia
e) Osteoporosis
15) Streptokinase contra-indicated,
a) Age > 75 years
b) Bronchial asthma
c) Background retinopathy
d) Atrial fibrillation
16) Poor prognostic factors in a patient with
chronic liver disease,
a) Elevated serum bilirubin
b) Low serum albumin
c) Elevated serum globulin
d) Prolonged INR
e) Elevated serum creatinine
17) Pleural effusion,
a) When mild, present as Kerley B lines
b) Of milky in nature is suggestive of
thoracic duct obstruction
c) Complication of pancreatitis
d) Congestive cardiac failure causes a
transudate
e) Of high protein content suggestive of
nephritic syndrome
18) Biochemical changes of chronic liver failure
due to chronic glomerular nephritis,
19) Following therapeutic measures are T/F?
a) Paracetamol should be avoided in
chronic liver failure
b) Nifedine is useful in the management
of chronic heart failure
20) Features of Chron's disease,
a) Ulcer limited to mucosa
b) Crypt abscesses
c) Intestinal fistula
d) Uveitis
e) Toxic megacolon
MFSU/MSWS 19
FACULTY OF MEDICINE
UNIVERSITY OF COLOMBO
FINAL MBBS EXAM – JUNE 2008
2001 A/L BATCH
MEDICINE
MFSU/MSWS 20
11) WOTF drugs and the adverse reactions c) Normal urine output
caused by them are correctly matched, d) Elevated jugular venous pressure
a) Carbamazepine – Diplopia e) Proteinuria of > 3 g/24 hrs
b) Carbamazepine – Steven Johnson 17) WOTF are characteristic features of
Sydrome oligouric acute renal failure
c) Phenytoin – Lymphadenopathy a) Hypokalemia
d) Phenytoin – SLE like syndrome b) Elevated blood pressure
e) Sodium valproate – Liver cell c) Acidosis
necrosis d) Pulmonary edema
12) T/F regarding iron, e) Increased serum creatinine
a) Absorbed in he terminal ileum 18) Arthritis is seen in WOTF,
b) Absorption is increased when a) Rubella
deficient b) Varicella zoster
c) Ferritin and hemosiderin are c) Chlamydia urethritis
transporters d) Chikungunya
d) Albumin is the transporter e) Hepatitis A
13) WOTF are T/F regarding gastroesophageal 19) T/F regarding Still’s Disease
reflux disease (GERD), a) Neutropenia is characteristic
a) Smoking is known to aggravate the b) Commonly involves the kidney
condition c) Majority is rheumatoid factor
b) Occur due to the reduction in the positive
esophageal sphincter pressure d) Serum ferritin is characteristically
c) Presents with iron deficiency elevated
anemia e) Fever lasting for more than 3
d) Dysphagia is a complication weeks makes this diagnosis
e) Proton pump inhibitors (PPIs) are unlikely
contra indicated 20) T/F regarding ankylosing spondylitis
14) WOTF are features of chronic liver cell a) Onset is around the fourth decade
disease b) Amyloidosis is a complication
a) Gynecomastia c) Causes a restrictive pattern in on
b) Dyputren’s contracture lung function testing
c) Clubbing d) Aortic stenosis is an association
d) Pruritus 21) WOTF is T/F regarding beta thalassaemia
e) High aminotransferase levels trait,
15) Renal biopsy is indicated in WOTF a) Causes a hypochromic microcytic
conditions blood picture
a) Diabetic nephropathy b) A normal Hb level excludes the
b) Acute renal failure with suspected diagnosis
SLE as the causes c) Splenomegaly is an expected
c) Asymptomatic proteinuria of > finding
2g/24 hrs d) Iron overload is a complication
d) Renal failure with small kidney e) Blood transfusion should be carried
e) Nephrotic syndrome between 2 – out if the Hb level falls below
10 years of age 9g/gL
16) WOTF is suggestive of Nephrotic than 22) WOTF features favour Vit B 12 deficiency
Nephritic syndrome in a patient with facial than folate deficiency in a patient with
puffiness megaloblastic anemia
a) Presence of hematuria a) Brisk reflexes in upper and lower
b) Normal blood pressure limbs
MFSU/MSWS 21
b) Glossitis e) Hemiplegia – cerebral abscess
c) Presence of intrinsic factor 28) Features of tuberculoid leprosy includes
antibodies a) Thickened peripheral nerves
d) Rising reticulocyte count following b) Skin nodules
parenteral Vit B 12 c) Anaesthetic patches in skin
e) Treatment with methotrexate in the d) Abundant mycobacteria in lesions
recent past e) A positive lepromin test
23) Features of haemolytic anemia includes 29) During anti tuberculous chemotherapy
a) Deep jaundice a) Rifampicin should be stopped if the
b) Hypochromasia patient develops jaundice
c) Reticulocytosis b) Colour blindness is side effect of
d) Increases proportion of direct ethambutol
bilirubin fraction c) Isoniazid induced hepatotoxicity is
e) Elevated serum LDH level more likely in malnourished
24) In diabetic neuropathy persons
a) Insulin is effective in improving d) Streptomycin is contraindicated in
diabetic amyotrophy pregnancy
b) Amitryptiline is an effective drug e) The commonest cause of peripheral
in painful neuropathy neuropathy is pyrazinamide
c) Antioxidants are known to improve 30) WOTF are features of metabolic syndrome
peripheral neuropathy a) Obesity
d) Fludrocortisone is effective in b) Hypoertension
autonomic neuropathy c) IHD
e) Gabapentine is effectivein painful d) Isolated hypertriglyceridemia
neuropathy e) Fatty liver
25) Hyperosmolar non-ketotic state 31) T/F regarding common krait bite
a) Is complication is type 1 DM a) Commonly seen in North central
b) Is characterized by severe province
dehydration b) Is an outdoor biter
c) Has a better prognosis than diabetic c) Causes coagulopathy\
ketoacidosis d) Painful local reaction
d) Has resistance to insulin e) Acute tubular necrosis
e) Predisposes to thrombosis 32) WOTF antidotes are posions/drugs are
26) In acromegaly correctly matched
a) Inferior temporal quadrantanopia is a) Desferrioxamine – iron
seen b) Flumazenil - benzodiazepine
b) Soft tissue hypertrophy is a feature c) N – acetylcysteine – paracetamol
c) Goiter is seen d) Pralidozime – Paraquol
d) Suppressio of growth hormones by e) Naloxone – TCAD
bromocriptine occurs 33) T/F regarding [aracetamol overdose
e) Decreased size of the frontal a) If presenting after 24 hours n –
sinuses is a feature acetylcysteine is not given
27) WOTF associations are correct regarding a b) Paracetamol levels done during
patient presenting with fever and headache first 4 – 16 hours is useful
a) Cervical lymnphadenopathy – c) Elevated PT – INR indicated bad
pneumococcal meningitis prognosis
b) Soft splenomegaly – typhoid d) Activated charcoal is helpful to
c) Mild icterus – malaria decrease absorption
d) Papilloedema – dengue fever
MFSU/MSWS 22
e) Liver transplantation is contra- a) Meiosis occurs in all cells
indicated b) Mitosis leads to half the number of
34) Hypoglycaemia is caused by chromosoes
a) Acute liver failure c) G0 is the resting phase
b) Alcohol intoxication d) In S phase, nuclear material
c) Phaeochromocytoma doubles
d) Addison’s disease e) Translocation of genetics material
e) Cerebral malaria occurs in mitosis
35) T/F regarding the treatment of 37) WOTF nail changes are correctly matched
thyrotoxicosis with the disease condition
a) Combination of carbimazole and a) Nail pitting – psoriasis
thyroxin is a recognized treatment b) Leukonychia – thyrotoxicosis
b) If remission doesn’t occur c) Half half nails – chronic renal
carbimazole is given for life failure
c) When on carbimazole, sore throat d) Oncholysis – liver disease
is an indication to do a FBC e) Koilonychia – Fe deficiency
36) T/F regarding cell division
MFSU/MSWS 23
FACULTY OF MEDICINE
UNIVERSITY OF COLOMBO
FINAL MBBS EXAM – DECEMBER 2009
2003 A/L BATCH
MEDICINE
MFSU/MSWS 24
d) Jaundice/bleeding tendency – c) Is a good indicator of acute
Hepatorenal syndrome blood loss
10) Clinical and biochemical features of d) Is expressed as a percentage
minimal disease e) Drops in the recovery phase of
a) Proteinuria > 3 mg/24 hrs DHF
b) Gross edema 13) S. osmolality 350 (increased) and U.
c) Microscopic hematuria osmolality is 200. What may be the
d) High LDL cholesterol possibilities?
e) Venous thrombosis a) Central DI
11) Regarding narcolepsy b) Nephrogenic DI
a) Sleep paralysis c) DM
b) Cataplexy d) Priary polydipsia
c) Uncontrollable sleep episodes e) CRF
d) Hypnagogic hallucinations 14) Leukoerythroblastic response seen in
12) PCV a) Sever sepsis
a) Measured by the capillary b) Cytotoxic drugs
method is higher than the c) Severe haemolysis
venous value d) Prostatic CA
b) Is elevated in babies born with e) Viral infection
IUGR
MFSU/MSWS 25
FACULTY OF MEDICINE
UNIVERSITY OF COLOMBO
FINAL MBBS EXAM – DECEMBER 2010
2004 A/L BATCH
MEDICINE
MFSU/MSWS 28
FACULTY OF MEDICINE
UNIVERSITY OF JAFFNA
FINAL MBBS EXAM – AUGUST 2010
2004 A/L BATCH
MEDICINE
MFSU/MSWS 29
12) Causes of delirium include severe infection a) Scaly patches with central clearing is
a) Increases thiamine seen in the tinea corporis
b) Hypoglycaemia b) Satellite lesions are seen in tinea
c) Abrupt withdrawal of lithium corporis
d) Severe stress c) Dermatophyte infection spread by
13) Complete heart block direct contact
a) Widen pulse pressure d) A fungal skin infection that has been
b) Produce cannon wave modified by topical steroid
c) Regular ventricular rate application is called tinea incognito
d) Associate with atrial fibrillation e) Oral itracoinazol is the drug of choice
produced irregularly irregular pulse in tinea capitits
e) Mostly due to inferior MI than 20) In treatment of hypothyroidism
anterior MI a) Carbimazol is contraindicated in
14) Obesity seen pregnant patients
a) Cushing’s syndrome b) Radioactive iodine treatment is not
b) PCOS recommended in grave’s disease
c) Addison’s c) Radioactive iodine treatment is
d) Grave’s effectively monitored by the
e) Prolactinemia estimation of TSH level in blood
15) Regarding ARF d) Radioactive carbimazol is preferred to
a) Haemolysis is mandatory carbimazol in toxic multi nodular
b) Spontaneous resolution occurs when goiter
due to ATN e) With carbimazol therapy monitoring
c) Ca Gluconate IV promotes recovery of WBC cannot be used to detect
d) Patient should be confined to bed rest neutropenia
e) NSAID is recommended for pain 21) Loos of tendon reflexes is characteristic of
relief a) Motor neurone disease
16) Recognized manifestation of acute nephritis b) Vit B12 deficiency
(AGN in) c) GBS
a) Diabetic nephropathy d) Myasthenia Gravis
b) Amyloidosis e) Transverse myelitis
c) HSP 22) ST depression is seen in
d) SLE a) Pericarditis
e) Minimal change disease b) Digoxin treatment
17) Causes of chronic diarrhoea c) Unstable angina
a) H. pylori infection d) LVH
b) Rota virus e) LV aneurysm
c) Metformin therapy 23) Septic arthritis commonly caused by gram –
d) carcinoid syndrome than gram + bacteria
e) Lactose intolerance a) Involve multiple joints
18) Leptospirosis b) Characterized by painful joint a
a) Is caused by gram + ve bacilli limitation of movement
b) Causes thrombocytopaenia c) Accurately diagnosed by joint
c) Causes ill defined lung shadows in radiography
CXR d) Effectively treated with intra articular
d) Is best treated with gentamycin antibiotics
e) Causes aseptic meningitis 24) Correctly paired
19) Regarding fungal infections of the skin a) People in the street talking about me –
Grandiose delusion
MFSU/MSWS 30
b) I hear a voice describing my actions – c) Herpes zoster in multiple dermatomes
through echo d) Atypical mycobacterial infection
c) My next door neighbour is planning to e) Kaposi sarcoma
kill me – delusion of reference 30) Potent reversible causes of cognitive
d) I hear voices ordering me not to eat – impairment
3rd person auditory hallucination a) Lewy body disease
e) My thoughts known to others – b) Hypothyroidisim
though broadcasting c) Hyponatremia
25) 26 year old boy is diagnosed with DM. It is d) Hepatic encephalopathy
more likely to be type 2 than type 1 if he has, e) Parkinson’s disease
a) Strong family history of DM 31) In the treatment of ascites due to cirrhosis
b) BMI – 26 a) Bed rest is mandatory
c) Rapid loss of 5 kg at time of b) Salt restriction is the principle cure for
presentation hyponatremia
d) Ketoacidosis on presentation c) Use if furosemide is known to worsen
e) Acanthosis nigricans hepatic encephalopathy
26) A 17 year old 50 kg previously healthy man d) FFP given a better therapeutic
admitted with drowsiness, on examination , response than stored plasma
froth in the mouth, PR 50 bpm, RR – 80 pm, e) Large volume abdominal paracentesis
BP – 70/50 mmHg, GCS 8/18 , ;lung aus- should be combined with albumin
diffuse crepitation, pupil – pinpoint. infusion
Followinf are T/F, 32) Regarding rheumatoid arthiritis
a) Gastric lavage done immediately a) Distal interphalangeal joint
b) History suggestive an involvement is a characteristic feature
organophosphate ingestion b) Restricted pattern if lung function is
c) 0.9% normal saline should be started due to lung parachyal disease
d) IV atropine bolus should be given c) Thrombocytosis indicate active
e) Should be intubated immediately disease
27) Myocarditis T/F, d) Episcleritis is complication
a) Specific ECG changes are present e) Pitting of nail is characteristic
b) Sudden cardiac arrest occur in young manifestation
adult 33) 70 year old male presented with tremor and
c) Viral infection is a known cause difficulty in walking which favour
d) Troponin are (+) Parkinsonism
e) Self limiting in majority a) Intention tremor
28) Regarding DM nephropathy b) Unilateral tremor
a) Absence of protein in heat coagulation c) Prediominantly affected the lower
test exclude DM nephropathy limb
b) ACEI uses delay the progression of d) Broad gait base
the illness e) Rapid response to levodopa
c) Most of patient need biopsy for 34) Regarding pulmonary TB
diagnosis a) BCG gives protection from post
d) Blood urea nitrogen irreversibly primary Tb
elevated b) Miliary Tb treated for 6 months
e) Good glycemic control delay the onset c) Sputum positive after 2 months of
of nephropathy treatment highly suggestive of
29) In HIV patients resistant organism
a) Oesophageal cadndidiasis 35) 23 year old gentlemen presented dengue for 4
b) Toxopolasmosis of the brain days duration. Cutaneous features – cold and
MFSU/MSWS 31
clammy peripheries, PR 1410/min, BP – 40) Acute lymphoblastic leukemia
90/80 mmHg, Sx platelets – 30000, Hb a) Most tumour cells derive from B cells
14g/dl, leukopenia , PCV – 52% line
a) Present initially resuscitate with 40 b) CNS involvement indicates poor
R% dextran prognosis
b) Initial infusion rate 5 mnl/kg/hr c) WBC count more than 50 x 107 /L
c) Platelet transfusion is indicated indicates poor prognosis
d) Increament of PCV due to leakage d) Monoarthritis is a feature
e) After 4 hours PCV – 45 % condition e) CXR shows mediastinal mass
remains the same following
resuscitation with normal saline
36) Features associated with high mortality in
pneumonia include
a) PR > 30 min
b) PR of 90/min
c) BU of 17 mmol/L
d) Mental confusion
e) Blood culture positive
37) A newly married female of 20 ys present with
dysuria, frequency of micturition and
suprapubic pain, WOTF T/F,
a) Amoxicillin is effective with
treatment,
b) Needs extensive evaluation of the
urinary tract for structural abnormality
c) Treated with vaginal estrogen cream
d) Alkaline urine suggestive of infection
with proteus
e) The patients sexual partner should be
investigated
38) Following are found in hyperactivity
disorders,
a) Its onset may occur before 36 months
b) Equal in male and female
c) Hyperactivity found only in home
d) Co morbidity associated with
hyperactivity disorders
e) Methylphendylate is useful in
treatment
39) T/F DHF
a) It does not occur in the age of 6/12
b) A negative Hess test exclude the
diagnosis
c) Bradycardia acute convalescence
d) Hyperosmolar fluid are preferable to
iso –osmolar fluid in shock
e) DIC is a complication
MFSU/MSWS 32
FACULTY OF MEDICINE
UNIVERSITY OF COLOMBO
FINAL MBBS EXAM – NOVEMBER 2012
2007 A/L BATCH
MEDICINE
pain on exertion. What is the next step
SBR of investigation for diagnosis of his
1. 63 year old diabetic and condition,
hypertensive woman presenting with a) Coronaryangiography
headache, anorexia and fatigue for 3 b) Echocardiograph
weeks duration. She developed c) Stress ECG
sudden onset of left sided facial, upper d) Troponin
limb, lower limb weakness. On e) ANP level
examination left side facial muscles, 5. 65 year old male was admitted with
UL/LL UMN weakness was present. community acquired pneumonia and
Investigations revealed ESR- 98 started on antibiotics. Next day he
mm/hr, WBC- 9000, Hb - 10, NCCT- developed palpitatiorns and pulse was
hydrocephalus and minor cortical irregular. BP= 110/80. What is the
infarcts, most possible diagnosis is, most likely cause?
a) Athermoatous stroke a) Atrial flutter with 2:1 block
b) TB meningitis b) Atrial fibrillation
c) SLE c) Sinus tachycardia
d) Giant cell arthritis d) SVT
e) Polyarthritis nodosa e) VT
6. 58 year oid male with hypertension
2. A 40 year old farrmer presenting and heart failure has breathlessness on
with acute lower limb weaknes, exertion. PR-80/min. BP-
following acute flu like illness. On 110/80mmHg. He is on furosemide
examination his lower limbs were and captopril. What is the next drug
paralyzed, reflexes were absent, that needs to be started?
extensor plantar response, bladder was a) HCT
palpable. What is the most likely b) Atenolof
diagnosis? c) Enalapri!
a) Guiline-Barre syndrome d) Metoprolol
b) Transverse myelitis e) ISMN
c) Anterior spinal artery thrombosis 7. 25 year old presenting with fever,
d) CIDP headache, myalgia, dry cough and
e) Poliomyelitis progressive difficulty in breathing for
3. Most significant feature suggestive 1 week duration. On examination
of idiopathic Parkinsonism, there are bilateral crepitations, liver is
a) Asymmetrical tremor palpable 1cm below costal margin and
b) Facial muscle weakness tender.
c) Unilateral sensory impairment WBC 4500 N-50% L-40%, ESR- 84,
d) Bradykinesia Na- 130, creatinine- normal, ALT-
e) Urinary incontinence 300. Likely diagnosis?
4. A 45 year old man present with a) Chronic bronchitis
recurrent episodes of left sided chest b) Community acquired pneumonia
MFSU/MSWS 33
c) Bronchopneumonia d) Olanzapine
d) Atypical pneumonia e) Quetiapine
e) Disseminated T8 33.51 year old man who is diagnosed
17. A 48 year old male presented with as having chronic alcoholic liver
polyuria, polydypsia and balanitis. A disease, presented with a history of
definitive diagnosis-of diabetes abdominal distension and ankle
mellitus is established by oedema. On examination he was
a) FBS 116 mg/dl febrile, icteric and mild abdominal
b) HbA1c 6.0% tenderness was there with an ascites.
c) PPBS 218 mg/dl Which of the following step is best to
d) Urine glucose ++ find out the cause for his
e) RBS 140 mg/dl presentation?
20. 67 year old male, a long standing a) Blood culture
rheumatoid arthritis patient on b) Peritoneal fluid aspiration
methotrexate, prednisolone, celecoxib, c) Full blood count
presenting with lethargy and anorexia, d) Urine full report
oral ulcers. Hb 9.1 g/dL, MCV 101.2 e) USS abdomen
fL, platelets 90000, WBC 4600 34. A 45 year old female diagnosed
mm'(Neut 1500) What is the most with severe iron deficiency anaemia
appropriate therapeutic option, has taken a high protein diet with iron
a) Folic acid and now her Hb level is 8.5g/dL.
b) Packed Cell tranfusion What is the next best step in
c) Buffy coat tranfusion management?
d) Ferrous sulphate a) Oral iron
e) Parenteral vit B12 b) Parenteral iron
24. Patient presents with recurrent left 35. A set of figures (marks of 7
sided chest pain on exertion. Resting students) is given. What is the best
ECG is normal. What is the next step indicator of its central tendency?
in his management? a) Mean
a) 2D ECHO b) Median
b) Exercise ECG c) standared deviation
26. Case scenario suggestive of d) standard error of mean
benzodiazepine overdose 36. A medical student falls asleep
(grandmother is taking treatment for during the lecture and gets up hearing
insomnia). 26 year old female took someone calling hiş name. What is
handful of drugs. On admission - this phenomenon called?
drowsy, RR 8/min a) Hypnogogic hallucination
a) Give iv flumezenil b) Hypnopompic hallucination
b) Insert an oropharyngeal airway TOF
c) Perform gastric larvage 6. Restrictive pattern of lung functions
30. 37 year old women diagnosed as are seen in
having schizophrenia on a) Ankylosing spondylitis
antipsychotics for 6 month duration. b) Asbestosis
She gained weight 20kg for last 6 c) Fibrosingalveolitis
months. Which one of the following d) Emphysema
drug is she most probabły on? e) Tracheal obstruction
a) Aripipazole 34. Which of the following result in
b) Halopiridol clinical cranial nerve palsies?
c) Ziprasidone a) TB meningitis
MFSU/MSWS 34
b) nasopharyngeal carcinoma
c) lateral medullary syndrome
d) cervical spondylosis
e) diabetes mellitus
10. T/F regarding SIADH
a) Hyperkalaemia
b) Plasma osmolaity is reduced
c) 5% dextrose is given in management.
11. Regarding body fluid volume
control,
a) Controlled by Sodium in the body
12. Regarding hypertension
a) Spironolactone is used to treat resistant
hypertension
13. Farmer who has ingested an
insecticide, presents with pinpoint
pupils. Regarding the management,
a) give atropine
b) give naloxone
c) tachycardia
MFSU/MSWS 35
FACULTY OF MEDICINE
UNIVERSITY OF COLOMBO
FINAL MBBS EXAM – NOVEMBER 2013
A/L BATCH
MEDICINE
radiates to neck, which of the following ix
would help in diagnosis ?(SBR)
a) Contrast CT
SBR b) EEG
1) 25 yr old girl, anemia", c) Echo
splenomegaly+, MCV =100, BP- red cell d) ECG
agglutination+, Unconjugated bilirubin- e) CXR
high, Urine urobilinogen- high, Urine
bile- high, What is the most important 6) 26 yr old female with a Hx of
investigation to arrive at a diagnosis? palpitions, past Hx of M, conscious, BP-
a) Coombs test 110/900 mmHg, ECG- broad QRS
b) Osmotic fragility complexes. What is the next
c) Retic count intervention?(SBR)
2) Diarrhea a) IV amiadarone
a) Staphylococcus aureus b) IV digoxin
3) 17 year old male with fever and c) IV adenosine
headache for 2 weeks, HB- 10, wbc-4000, d) Carotid massage
plt- 14000, EEG- normal, ESR- 80, CSF- e) DC shock
wbc -high, głucose - low, protein -high, 7) 70 yr old DM, HT,RA, on several oral
lymphocytes -high, what is the next step drugs, knee joint pain and swelling,
in management (department) obese+, Ex – mildly swollen, mild joint
a) Anti TB drugs with steroids effusion, non-tender, no warmth, H8-9,
b) IV acyclovir wbc-NL, ESR- 40.most likely cause for
c) Ceftriaxone with steroids presentation?
d) V ampicillin a) Gout
e) Oral artemesinin+lamefrentine b) OA
4) 35 year old male, with abdominal c) Prepatellar bursitis
obesity, acne, hirsutism, bp-150/95, FBS d) Septic arthritis
201mg/dl , k+ =2,3 what is the next most e) Synovial seroma
appropriate investigation to 8) A 26 year old male presented with dry
diagnose?(department) cough and difficuity in breathing for 1
a) CT adrenal week duration. ESR-40. Fever + WBC
b) pituitary MRI 9000 Chest Xraybilateral infiltrates. What
c) dexamethasone suppression test is the most likely diagnosiş? (department)
d) random cortisof a) Mycoplasma pneumonia
e) ACTH level b) Strep. Pneumonie pneumonia
5) A 79 yr old female presented with a Hx c) TB "*
of recurrent falis and transient LOC, on d) ldiopathic fibrosing alveolitis
ex- no neurologicat signs ,BP- 110/90, 9) 32 yr old pt with 4 days cough and
pules 75, on auscultation there was a fever 4 days prior to illness visited a
systolic murmur in aortic area which friend in hospital he has minimal lung
signs chest X ray B/L patchy shadows,
MFSU/MSWS 36
most likely etiological agent? 14) Respiratory distress, restless,
(department) tachycardia, tachypnea, low bp, stridor
a) Pneumococcus with cyanosis
b) Pseudomonas a) Acute severe asthma
c) Staphaureus b) Hysterical hyperventilation
d) Legionella c) Foreign body
e) Mycoplasma d) Pneumonia
10) 70 year old male, known pt with e) Pneumothorax
asthma presents with SOB,bilateral T/F- Medicine
rhonchi, crepts. He had undergone a 1) osteoarthritis occurs as a complication
CABG3 years back and an echo 1 month of
back. EF 35% (department) Ix of choice a) rheumatoid arthritis
to differentiate asthma from acute LVF b) perthes disease
a) BNP c) Rheumatic fever
b) CXR d) Haemarthrosis
c) Echo e) Septic arthritis
d) ECG 2) Malaria
e) PEFR a) primaquine is essential for p.vivax
11) 68 year old female presented with b) Artemathazine is given in pregnancy
headache, drowsiness, fever for 2 days. c) Antigen detection is more specific than
She has a history of recurrent ear 3) t/f regarding inherited disorders
intections. Most important Ix for Dx? a) AR-has no carriers
a) Ear swab b) XR-clinically occurs in males
b) Blood culture c) Adult PCKD is XD
c) CT brain d) Mitochondrial disorders inheritance is
d) ESR paternal
e) Lumbar puncture e) Ad- 50% of offspring's get the disorder
4) t/f which of the following inhibit K+
12) 47 year old male taking excess secretion?
quantities of alcohol presented with a) High K" levels in biood
epigastric pain and vomiting. There is b) Aldosterone inhibitors
radiation of pain to back. Examination c) Diuretics
reveals a BP=90/60 and diffuse mild d) Metabolic alkalosis
epigastric tenderness with no e) High Na intake
organomegaly. What is most likely
diagnosis? 5) T/f which of the following causes
a) Gastroesophageal reflux cirrhosis as a consequence or
13)55 year old driver with DM for 10 complication?
years presented with worsening nocturia. a) Hep E infection
He was onmetformin 500mg bd and b) Alcoholic steatohepatitis
glibenclemide 10mg bd. He had past hx c) Acute PCM poisoning
of Mi and LV dysfunction. 6) T/F
FBS-320mg/dl, HbA1c a) Liver receives 80% of blood supply by
=10.2%, S.Cr=32g/di) hepatic artery
a) Start on insulin b) Encephalopathy in apt with cirrhosis
b) Add pioglitazone indicate poor prognosis
c) Reduced calorie intake c) Commonest cause for cirrhosis in Sri
Lanka is Hep B
MFSU/MSWS 37
7) T/F. Glomerular pathology should be 14)
suspected when? a) 02
a) SLE+,with hematuria b) Salbutamol nebulization
b) S0yr DM sterile payuria c) pratropium bromide
c) male with colic d) V hydrocortisone
d) i
e) 14 yr, 2days hx of painful hematuria
8) T/f which of the following are
recognized cases of pulmonary
hypertension?
a) Obstructive sleep apnoea
b) Mitral Stenosis
c) Pericardial effusion
d) COPD
e) Recurrent pulmonary embolization
9) Which of the following are true?
a) Psychogenic breathłessness mainly
disturbs sleep
b) Thromboembolism alone caúses chronic
breathlessness
10.T/f 78 yr old man sudden onset black
outs while exercising at gymnasium lasted
2 minutes were reviled by cardiac
massage,which of the following are
appropriate in investigating to diagnose?
a) Plain CT
b) ECG
c) ECHO
d) Ex- { ECG
e) Halter monitoring
11) Distal convoluted tubular K+
secretion is reduced by
a) Hyperkalemia
b) A diuretic acting on proximal tubule to
reduce Na+ absorption
c) Tremor
d) Diabetes insipidus
e) Cardiac arrhythmia
12)
a) FBC
b) S. free TSH
c) S. TSH
d) US thyroid and FNAC
13) Delirium
a) Acute onset is characteristic.
b) Confirm only by CT brain
c) Is associated with hyper-reflexia
d) treated with antipsychotic at least 5
months
MFSU/MSWS 38
FACULTY OF MEDICINE
UNIVERSITY OF COLOMBO
FINAL MBBS EXAM (MAIN) – NOVEMBER 2014
2008 A/L BATCH
MEDICINE
MFSU/MSWS 39
9) 68 year old male presented with past c) Polyarteritis nodosa
history of TB (completed Treatment with d) PCKD
bronchiectasis, this time presented with fever and e) CRF
cough. Temp - 38.3 C. SpO2 - 88%.what is the
best empirical antibiotic for this man? a) Benzyl 14) 16 yr old boy presented with facial
penicillin puffiness, ankle oedema. Urine dipstick +++, 24 hr
b) Oseltamivir urine protein- 4.5g. what is the most likely
c) Clarithromycin diagnosis? a) Ig A nephropathy
d) Tetracycline b) Polycystic kidney disease
e) Ceftazidime c) Minimal change nephrotic Xd
d) Post infectious GN
10) 68 year old patient is on warfarin e) RPGN
presented with hematemesis. His INR is 9.what is
themost appropriate management a) 15) Which of the following is the best
Cryoprecipitate indicator of chronic kidney disease?
b) FFP a) Blood urea > 110
c) IV vitamin K b) Bilateral shrunken kidneys
d) Omeprazole c) Creatinine clearance - low
e) Endoscopic banding d) proteinuria
e) serum K+ - 6.5 mmol/L
11) 28 year old male presented with watery
diarrhea for 6 weeks duration with loss of 16) 25 year old woman presenting with a
weightLOA, abdominal pain and low grade fever, history of excessive sweating, loss of weight inspite
O/E there was a right iliac fossa mass. What is the of good appetite and palpitations. O/E - diffuse
most suitable diagnosis, a) Coeliac disease enlargement of thyroid gland, pulse 120 bpm. What
b) Giardiasis is the most appropriate Investigation in initial
c) Intestinal TB management? a) Free T3
d) Irritable bowel syndrome. b) UST of the neck
e) Ulcerative colitis c) Thyroid peroxidase Ab
d) TSH receptor antibody
12) 54 year old known patient with alcoholic e) Thyroid technetium scan
cirrhosis, presented with altered sleep
pattern,ascites and ankle oedema. O/E his 17) 15 yr old boy presented with polyuria and
abdomen is tensed and tender. FBC - Hb-11g/dl, weight loss. On admission RBS - 500 mg/dl,pH 7.2,
WBC -15,000(N-80%), PLT 150,000. Ascitic HCO3 - 14 mEq/L. his blood sugar was under
fluid-450/mm3 of neutrophils. What is the best control after few days of hospital stay. Most
management for this patient? a) Cefotaxime appropriate management on discharge, a) metformin
b) Frusemide b) once a day long acting insulin
c) Salt poor plasma c) glibenclamide
d) Large volume paracentesis d) metformin with glibenclamide with bed
e) Low salt diet time basal insulin
e) short acting insulin before main meals
13) 32 year old male with HTN presented to and long acting insulin at night
ward. His father died due to SAH. UFR - Alb
RBC 4050. S.Cr- 112. Na+ - 139.K+ - 4.2 Hb - 16 18) 67 yr old female who is a known patient
g/dl. HCT-45. WBC- 7000. pH - 7.4. ESR -5 24 with DM, on oral hypoglycemic agentspresented
hour protein excretion - 900 mg what is the most with RBS - 455 mg/dl. O/E she was dehydrated, PR-
likely diagnosis? a) Renal artery stenosis 100 bpm, BP - 100/70mmHg. Which of the
b) Nephrotic Xd
MFSU/MSWS 40
following is the first step in management? a) Give b) Ciprofloxacin
s/c soluble insulin c) Clarithromycin
b) Give iv normal saline d) doxycycline
c) Give 0.45% saline e) metronidazole
d) Increase the dose of oral hypoglycemic
drugs 23) 24 year old male presented with
e) Catheterize the patient headache, myalgia, arthralgia, Intermittent fever
with chills for 1 week. He has returned from India 2
19) 65 yr old male presented with pallor and weeks back. O/E - mild hepatomegaly with no LN
lethargy. O/E-he is pale, generalized enlargement. FBC-Hb-10.8 g/dl WCC- NL, PLT -
lymphadenopathy, 3cm hepatomegaly and 4cm 120,000 (150,000-450,000).What is the most
splenomegaly. Ix-Hb-9g/dl, WBC-40,000, Pit appropriate Initial investigation to come into a
- 100,000. Lymphocytes 90%. What is the diagnosis? a) Blood culture
most likely diagnosis, b) HIV antibody level
a) Non hodgkin lymphoma c) Urine microscopy
b) CLL d) Thick blood film for malaria parasite
c) CML e) Standard agglutination test (SAT)
d) Myelofibrosis
24) 34 yr old farmer was poisoned with
20) 23 yr old boy presented with anaemia 0/E organophosphate is treating with i/v atropine and
- he is pale, no organomegalyInvestigations pralidoxime. On the 2 day he was flushed. He has
revealed - Hb -9.5g/dl, MCV-60, MCHC-16, fever, PR-120 and BP-160/100 mm Which of the
WBC -6,000, PLT - 200,000. S. ferritin – following is the best treatment of choice? a) IV
275(40200) What is the most likely diagnosis, a) fluids
Thalassemia trait b) Stop pralidoxime
b) Beta thalassemia major c) CPAP
c) Iron deficiency anaemia d) Stop atropine
d) Pernicious anaemia e) Give metronidazole
e) Myelodysplastic syndrome.
25) 14 yr old boy after 10hr history of 6
21) 56 year old man presented with tender weeds of oleander poisoning presented to casualty
swollen 1" metatarsophalangeal joint. He was on ward. He is conscious and rational. C/o - vomiting
frusemide for ankle oedema, Investigation and abdominal pain. HR -35. What is the most
revealed Hb - 10.1g/dl, WBC - 19,000, ESR - 95, appropriate next step in Mx? a) Gastric lavage
Scr - 3.42. what is the most likely diagnosis, a) b) IV atropine
Gouty arthropathy c) IV isoprenaline
b) Cellulitis d) Give activated charcoal
c) TB arthropathy e) Start temporary cardiac pacemaker
d) Rheumatoid arthritis
e) Septic arthritis 26) 27 yr old male was bitten by a unknown
snake at 8 pm in the night. His R/ leg is swollen
22) 21-year-old boy from kataragama where the bitten mark is present. He complains of
presented with fever, myalgia, headache and pain in the leg. He is conscious and rational. PR 90 ,
tinnitus for2 days, ONE-conjunctival injection, BP - 110/70 mm Hg. What is most appropriate the
maculopapular rash, cervical lymphadenopathy first step in management a) Administer antivenom
and hepatomegaly was present. PLT -low. AST, immediately
ALT- elevated. What is the most appropriate b) NSAIDs for pain
antibiotic? c) Administer tetanus toxoid
a) Benzyl penicillin d) Treat with oral cloxacillin
MFSU/MSWS 41
e) 20min WBCT
MFSU/MSWS 42
FACULTY OF MEDICINE
UNIVERSITY OF COLOMBO
FINAL MBBS EXAM (MAIN) – NOVEMBER 2015
2009 A/L BATCH
MEDICINE
1) Regarding GBS 6) Exudative pleural effusions are seen in
a) Type 1 respiratory failure a feature a) Hypothyroidism
b) Global areflexia is typical. b) Meig syndrome
c) Distal demyelination is a feature c) Nephrotic syndrome
d) Elevated CSF protein is a feature d) Pancreatitis
e) Treated with IV methylprednisolone. e) Pulmonary infarction
2) WOF are true about myasthenia gravis 7) Which of the following investigations are
a) Muscle wasting is a prominent feature correctly matched with their cause for chronic liver
b) fluctuating muscle weakness is seen cell disease?
c) Heart is not affected a) Primary biliary cirrhosis - anti ds DNA
d) Proximal muscles are mostly affected b) Auto immune hepatitis - anti smooth
e) Associated with thymic hyperplasia. muscle antibody
c) Wilson's disease - 24 hour urinary copper
3) On examination BP 120/50 mmHg excretion
recorded in the right upper arm. What are d) Hepatitis B - HBsAg
thepossibilities a) AR e) Hemochromatosis -24 hour hemosiderin
b) MR excretion
c) AS
d) PDA 8) Causes of cirrhosis,
e) VSD a) Hepatitis E
b) After acute poisoning of methyl alcohol
4) Regarding arctic stenosis c) Asymptomatic gall stones without
a) Can present with angina common bile duct obstruction
b) ECG may show left ventricular d) NASH
hypertrophy e) Portal vein thrombosis
c) In severe disease there can be narrow
pulse pressure 9) A 35 year old male presents with haematuria of 1
d) Has a thrusting apex day duration. He has had a similar episode 1 1/2
e) Has a loud 2t heart sound. years back. He is on treatment for diabetes
mellitus.On examination there is no edema. Blood
5) Which is/are correct pressure is elevated UFR-RBC 200, pus cells 3-
a) Tidal percussion is positive in a 4,protein nil. No RBC casts and dysmorphic red
pneumothorax cells. Serum creatinine and blood urea normal,Which
b) Vocal resonance is reduced in a pleural of the following can be a cause?
effusion, a) Post streptococcal glomerulonephritis.
c) Cracked pot sign is seen in large lung b) UTI
cavities c) Renal cell carcinoma
d) Coin sign is seen in a pneumothorax d) IgA nephropathy
e) High pitched bronchial breathing is heard
over a consolidation 10) Metabolic acidosis is seen in
a) Severe vomiting
MFSU/MSWS 43
b) Severe diarrhea. 16) Which of the following cause a characteristic
c) Ketosis rash in palms and soles?
d) Diuretic abuse a) Secondary syphilis
e) Treatment with acetazolamide b) Herpes simplex virus
c) Condylomata accuminata
11) Polydipsia is known to occur in d) Keratoderma blennorrhagica
a) SIADH e) Arsenic poisoning
b) Psychogenic polydipsia
c) Hyperkalemia 17) Characteristics of autosomal dominant disorders,
d) CKD a) Abnormal genes from both parents are a
e) Hypokalemia prerequisite for the disease
b) Probability of affecting the siblings will
12) Regarding warm autoimmune haemolytic be determined by the affected 1" child
anemia c) Abnormal gene is dominant
a) Haemolysis occurs in peripheries d) Who do not have the disease can't pass
b) Associated with IgM antibodies the disease
c) Caused by Mycoplasma pneumoniae e) Huntington's disease is an example
d) is usually Coombs positive
e) Can be caused by methyldopa treatment. 18) Regarding GLP (Glucagon Like Peptide)
a) It is an incretin.
13) Regarding acute gout b) Produce by pancreas
a) Low dose aspirin lowers uric acid c) Increases insulin secretion
excretion d) Metabolized by DPP-4
b) Uric acid concentration reduces after an e) GLP-1 agonists can be used to treat
acute attack hypoglycemia
c) Allopurinol should be given within the 1
week of an acute attack 19) Transient psychotic disorders,
d) Common in 1 metatarsophalangeal joint a) Precipitated by stress
e) Responds well to NSAIDS b) No need of antipsychotic treatment
c) Resolve within 3 months
14) Correctly matched micro organism with d) Appear over 2 weeks
medication
a) Methicillin sensitive Staphylococcus - 20) Regarding deliberate self harm
cloxacillin a) Carried out repeatedly till they die.
b) Pseudomonas aeruginosa - co amoxiclav b) Commonly done using poisoning and
c) Mycoplasma pneumoniae- clarithromycin overdose drugs.
d) Plasmodium vivax - chloroquine c) Management should involve legal
e) Chlamydia trachomatis doxycycline authorities.
d) More common in females
15) Regarding malaria in Sri Lanka e) Majority do not have a psychiatric illness.
a) Is currently an endemic disease in Sri
Lanka 21) SBR
b) Vivax malaria is known as tertian malaria
c) Falciparum malaria can cause nephritic 22) A 49 year old male was found unconscious on
syndrome road. On arrival his temperature is 37°C, BP 160/90
d) Pathology of cerebral malaria is mmHg. PR-90 bpm. Cardiac auscultation findings
encephalopathy are normal. His RR-20/min, lungsclear. There is no
e) Artemisinin is the most effective drug neck stiffness. B/L pupils are 1mm in diameter,
poorly reactive to lightreflex and fundus is difficult
MFSU/MSWS 44
to examine B/L'Lower limb reflexes are normal JVP, bibasal crepitations and an enlarged liver. BP
except B/L plantar responses, which are 180/80 mmHg, RR - 25/min, PR36 BPM. OnECG
extensors.What is the most likely diagnosis? a) heart rate was 96 bpm.What is the best management
Snake bite envenomation option? a) Digoxin
b) Organophosphate poisoning b) Digoxin + Furosemide
c) Pontine hemorrhage c) Furosemide
d) Myasthenic crisis d) Abdominal paracentesis
e) Botulinum toxicity e) Transthoracic pacemaker
23) A 72 year old male was brought to the
medical clinic by his wife. Wife complains that 27) A 55 year old male with history of diabetes
her husband has forgetfulness and poor mellitus and hypertension presented with retrosternal
concentration during last few months. She also chest pain. ECG showed 4mm ST elevations in lead
complains of urinary incontinence and abnormal V1 to V6. Streptokinase wasngiven after excluding
walk for the last few months. On examination he contraindications. After 2 hours, pain persists and
is having an unstable gait. His fundal examination ECG shows 3mm STelevations. What is the best
is normal.What is the most possible diagnosis? management option? a) Urgent coronary artery
a) Lewy body dementia bypass grafting
b) Motor neuron disease b) Rescue percutaneous coronary
c) Normal pressure hydrocephalus interventions
d) Parkinsonism c) Repeat thrombolysis with alteplase
e) Alzheimer disease d) Subcutaneous enoxaparin
e) IV GTN
24) A 28 year old prisoner was brought to the 28) A chronic smoker developed cough and
medical ward with an acute fever episode. His progressive SOB over 10 years. He presented
CSF analysis findings were, Polymorphs 2, withexacerbation of symptoms for 1 week with
Lymphocytes 48,Protein 458 mg/dl,Sugar 28 productive cough. Physician ordered O2 via a face
mg/dl,RBS 200 mg/dl/ What is the most probable a) mask. What is the percentage, which you
diagnosis? a) Partially treated meningitis would give to the face mask?
b) Tuberculous meningitis b) 45%
c) Viral meningoencephalitis c) 35%
d) Leptospirosis d) 60%
e) Viral encephalitis e) 80%
f) 100%
25) A 55 year old male who is a diagnosed 29) 2A 23 year old man presented with mild fever for
patient with rheumatic heart disease, presented 2 weeks. CXR shows a right sided pleuraleffusion.
with drowsiness and faintishness. On examination Full blood count was done.
he has irregularly irregular pulse, PR-160 a) WBC-8000/mm3
bpm,and BP 70/50 mmHg. What is the most b) Neutrophils-60%
appropriate management for this patient? a) c) Lymphocytes-40%
Amiodarone d) Pleural fluid aspiration shows
b) IV dobutamine predominant lymphocytosis.What is the most
c) DC cardioversion likely diagnosis?
d) N/S intravenous infusion e) TB
e) IV digoxin f) Connective tissue disease
g) Lymphoma
26) A 60 year old previously well male h) Empyema
complained of progressive breathlessness, i) Parapneumonic effusion
abdominaldiscomfort and swelling of the legs. On 30) A 65 year old male with cough, cold and SOB for
examination there was gross ascites, elevated 3 years, presented with an acute exacerbation. He
MFSU/MSWS 45
was treated with salbutamol + 0, nebulization. e) Sildenafil
After 4 hours he developed confusion and
restlessness. What is the most appropriate 35) A 60 year old patient presented to medical unit
investigation to find out the causefor his acute due to an unidentified snake bite. Then he was
confusional state? a) FBC diagnosed to have acute kidney injury. His urine
b) CRP output was 600ml/24h. What is the best management
c) CXR option?
d) CT Brain a) Restrict protein intake up to 40g/day.
e) ABG b) Restrict fluid intake to daily loss
31) A 50 year old patient with cirrhosis c) Start 0.9% saline daily maintainance
presented with variceal bleeding, He was solution
managed with octreotide and banding What is the d) Give low caloric and low protein diet
most likely measure to prevent further variceal e) Give high caloric and high protein diet.
bleeding? a) Interval band ligation
b) Oral proton pump inhibitors 36) A 72 year old male on oral hypoglycemics
c) Interval sclerotherapy presents with fever for 3 days. Examinationshows
d) Oral nitrates dehydration and lethargy. Investigations show RBS-
e) Oral beta blockers 620 mg/dl, Na+ 150 mmol/l, K+ 5mmol/l,
HCO3_25, pH -7.35.What is the best next step in the
32) What is the best management option for management? a) soluble insulin 20mg IM
hepatorenal syndrome? b) 0.9% saline bolus
a) Reduce diuretic therapy c) Heparinization
b) IV fluid resuscitation with normal saline d) Increase oral hypoglycemics
c) IV terlipressin e) Start mixtard insulin
d) Repeated large volume paracentesis
e) IV ceftriaxone 37) A 35 year old female presented with recent onset
33) A 3 year medical student had an malaise, tremor and palpitations for 2weeks with
accidental prick injury to the hand while taking a tenderness of the neck.TSH <0.01 mmol/l (0.15-
blood sample of a patient, suspected with Hep B 3,5).T-0.35ng/dl, ESR-45mm/1 hourWhat is the most
infection. What is/are the most appropriate likely diagnosis? a) Grave's disease
investigations that should be performed b) Thyrotoxic crisis
subsequently? a) HBsAg of the patient c) Subacute thyroiditis
b) HBsAg of the student and HIV antibody d) Toxic multinodular goiter
of the patient e) Carcinoma of thyroid
c) HepBsAb of the student and HIV
antibody of the patient 38) 3A 55 year old male driver with diabetes mellitus
d) HBsAg and HIV antibody of the patient for 10 years as a past history of myocardialinfarction
e) HBsAg and HIV antibody in both & LVF. He is on metformin 500mg tds and
glibenciemide 10mg bd. He presentswith worsening
34) A 35 year old male presents with bilateral nocturia. FBS 320mg/dl. HbAlc>10%.What is the
ankle swelling to his routine medical checkup. BP best management option for him? a) Add insulin
160/105 mmHg, PR-78 bpm, UFR- protein ++, b) Low cholesterol diet
RBC- 2 hpf, serum creatinine-increased, blood c) Increase metformin dose
urea-increased, Nat- normal, K+-normal, GFR-40, d) Add acarbose
USS-increased cortical echogenicity,normal size e) Add pioglitazone
kidney.What will be the best drug? a) Methyldopa
b) Nifedipine 39) A 25 year old male presented with hypochromic
c) Captopril microcytic anemia and blood pictureshowed target
d) Prazosin cells Serum iron and ferritin levels are in upper
MFSU/MSWS 46
normal level. HbA1c level ielevated. What is the bpm and BP-160 mmHg. Which of the following is
diagnosis? the best treatment option? a) IV fluid
a) Hereditary spherocytosis b) Stop pralidoxime
b) Sideroblastic anemia c) Connect to CPAP
c) Beta thalassaemia trait d) Stop atropine
d) Normocytic normochromic anemia e) Give metronidazole
e) Alpha thalassaemia minor
44) In 18 year old girl presented following ingestion
40) A 20 year old female is tired and having of 20 tablets of paracetamol, After 3 hours she had
SOB. She had fever and cough 5 days ago and recurrent episodes of vomiting. Her weight is 45kg
wastreated with azithromycin 500mg dailyHb- What is the most appropriatemanagement? a) Gastric
8.3g/dl WBC-5000 PLT-180000 MCV-99 lavage
MCH28Monospot test-negative Reticulocyte b) Observe without any intervention
count-7%Blood film- Red cell agglutination What c) Check plasma paracetamol level
is the diagnosis? d) Give NAC
a) G6PD e) Give methionine
b) Mycoplasma pneumonia
c) DIC 45) A new diagnostic test was introduced for the
d) IMN disease X. specificity is 75% & sensitivity is 85%.
e) B12 deficiency Prevalence of the disease X is 59. How many
patients are not diagnosed by the test among a
41) An 80 year old male presented with 4 population of 1000? a) 50
weeks history of fever. During last 2 years he b) 42
hadsimilar lower urinary tract symptoms. c) 8
Symptoms disappear after treating with d) 4
norfloxacilin But fever persists. He has e) 15
hypertension, diabetes and prostatism and is on 46) A 20 year old girl presented with acute
metformin andprazosin. He has mild pallor and a exacerbation of bronchial asthma. Oxygen was
splenomegaly. A diagnosis of infective givenand was nebulized with salbutamol. But no
endocarditis is made.What is the most likely improvement.Next step in management? a) Repeat
causative organism for infective endocarditis in salbutamol
this patient? a) Enterococci b) CXR
b) Gram negative organism c) CPAP
c) Salmonella d) IV aminophylline
d) Staphylococci e) IV MgSO4
e) Viridans Streptococci 47) A 55 year old male presented to the medical ward
after being bitten by an unidentified snake. He was
42) Which of the following is the best for having right leg swelling up to the knee joint and
preventing hospital acquired infections, blackish discoloration atankle joint with an oozing
a) Give prophylactic antibiotics puncture mark. No other physical signs present
b) Vaccination of the hospital workers Clotting test is normal.What would be the immediate
c) Proper hand care of hospital workers decision?
d) Early discharge a) Antivenom and surgical referral
e) Isolation of the patients b) Surgical referral
c) Limb elevation & antibiotic
43) A 34 year old farmer was poisoned with d) Antibiotics, antivenom and surgical
organophosphate and is on treatment with IV referral
atropine and pralidoxime. On the 2 day he was e) Antibiotics and antivenom
flushed & agitated. He had dilated pupils,PR120
MFSU/MSWS 47
48) A 45 year old female presented with
abdominal pain of 2 years duration. Examination
was unremarkable. She has undergone several
endoscopies, colonoscopies and CT scans
whichare normal. Further she expressed that she
is in fear of having an intestinal carcinoma.What
is the most probable diagnosis? a) Depressive
disorder
b) OCD
c) Hypochondriasis
d) Anxiety disorder
e) Schizophrenia
49) A 30 year old accountant seeks medical
advice because he worries about an upcoming
publicspeech to be delivered in front of his
colleagues. He always feels uncomfortable in
such activities. Recently he has started to feel that
his boss is scrutinizing his work and now he finds
it difficult to sign cheques and other documents in
front of others. He knows that his thoughts are not
based on adequate grounds. Now he is avoiding
the canteen during lunchand started to take
alcohol.What is the most likely condition he is
having? a) Social phobia
b) Schizophrenia
c) Agoraphobia
d) Alcoholism
e) Panic disorder
MFSU/MSWS 48
FACULTY OF MEDICINE
UNIVERSITY OF COLOMBO
FINAL MBBS EXAM (MAIN) –2016
2010 A/L BATCH
MEDICINE
1) Regarding Myasthenia gravis, d) he is afebrile
a) Pupillary reflex is affected e) e)
b) Touch sensation of hand is affected 7) Regarding fat malabsorption,
c) Circulating anti cholinesterase enzyme a) Vitamin deficiency is uncommon
antibodies arehelpful in diagnosing MG b) Occurs in bacterial overgrowth
d) Muscle biopsy is needed for diagnosis c) Occurs in obstructive jaundice
e) Aminoglycoside improves fatigability d) Diagnosed when faecal fat content > 7 g in 24
2) 22-year-old girl complains of severe hour
throbbing headachesometimes lasting for as long e) 90% of exocrine pancreas function should be
as 24 hours, associated with vomiting, before fat malabsorption occurs
photophobia and phonophobia. WOTF drugs can 8) Regarding inflammatory bowel disease,
be used to treat these acute attacks? a) Pizotifen a) Crohn's disease is commonly seen in rectum
b) Amitriptyline b) Ulcerative colitis causes transmural ulceration of
c) Paracetamol the bowel wall
d) Sumatriptan c) Intravenous steroids are indicated in acute severe
e) Propranolol ulcerative colitis
3) Increased JVP, with normal waveform d) Colectomy will cure Crohn's disease
a) Obstructed SVC e) Granuloma on histology is characteristic of
b) Complete Heart block Crohn's disease
c) IV fluid overload 9) What are the characteristic features of minimal
d) Cardiac failure changen ephrotic syndrome?
e) Tricuspid regurgitation a) Elevated blood pressure
4) Drugs and their side effects 10) 58 years old male with type Il diabetes mellitus,
a) Sodium valproate hirsutism whose BMI is 30.4 and BP is 150/100mmHg has
b) Amiodarone - photosensitivity poor glycaemic control.His ejection fraction is 25%,
c) Propafenone - corneal microdeposits FBS level is 165mg/dl, HbA1c is 8.9, S.Cr is 3.1
d) Lignocaine - convulsions mmol/L, eGFR is 25ml/mg. What are thedrugs that
e) Digoxin - confusion he can use? a) Gliclazide.
5) Regarding diagnosis of pulmonary b) Tolbutamide.
tuberculosis, c) Pioglitazone.
a) Positive smear confirms the diagnosis d) Metformin.
b) Silver staining of acid fast bacilli e) Sitagliptin.
c) Culture report for AFB takes 6 days 11) Regarding thyrotoxicosis
d) Treatment is with IV cefotaxime a) commonly convert to malignancy
e) Positive gamma interferon test is b) Exophthalmos is characteristic in
diagnostic Hashimoto'sthyroiditis
6) A patient presenting with acute c) Can present with chronic diarrhoea
exacerbation of asthma can be discharged if d) Proximal muscle weakness is a well-known
a) stable on oral medication without feature
nebulization during past 48 hours e) Propylthiouracil is the drug of choice in 1st
b) no nocturnal wheezing trimester of 12) What are the causes for
c) normal chest x-ray megaloblastic anaemia?
MFSU/MSWS 49
a) Rheumatoid arthritis d) Paranoid delusion
b) Chronic alcoholism e) Vivid illusion
c) Pernicious anaemia 20) Which of the following can be used to suggest
d) Hypothyroidism dengue fever in first day of fever?
e) Liver cell disease a) Low platelet count
13) What are the features of osteoarthritis? b) High haematocrit
a) Increased ESR c) NS1 antigen
b) Crepitus over the joint d) Dengue DNA PCR
c) Osteophytes in the radiograph e) Dengue IgM antibody levels
d) Limb deformity 21) What are the features to detect entering the
e) Joint effusion dengue critical phase?
14) Which of the following have vaccines? a) Platelet count
a) Hepatitis A b) Hematocrit
b) Typhoid c) Ca2+ levels
c) Leptospirosis d) Urine output
d) Hepatitis C e) Pulse pressure
e) Haemophilus influenza infection 22) Amiodarone
15) Koebner phenomenon is seen in a) Can be used to treat SVT
a) Psoriasis b) It is known to cause hypothyroidism
b) Lichen planus c) It is known to cause hypoparathyroidism
c) Molluscum contagiosum d) SBR
d) Measles 23) A 59-year-old male presented with right sided
e) Arsenic poisoning spastic hemiparesis with increased reflexes, and right
16) Which of the following are disorders due side loss of proprioception and joint position
to abnormalitiesin chromosome number? sensation and left side loss of pain sensation. There
a) Down syndrome was no loss cranial nerve palsyand facial sensation
b) Haemophilia A was normal. What is the most likely place of lesion?
c) Klinefelter syndrome a) Left side lesion to cervical cord
d) Duchenne muscular dystrophy b) Lesion in right midbrain
e) Turner syndrome c) Right side lesion to cervical cord
17) Structures involved in light reflex, d) Lesion in left cerebral cortex
a) Occipital cortex e) Lesion in right cerebral cortex
b) Lateral geniculate body 24) 55-year-old male presented with recurrent
c) Edinger Westphal nucleus seizures. What is the first line investigation? a) FBC
d) Oculomotor nerve b) CECT brain
e) Ciliary ganglion c) Random blood sugar
18) Which of the following are true about d) Serum electrolytes
Lithium? e) EEG
a) It is used to treat bipolar depression.
b) Has a narrow therapeutic index 25) A 55-year-old presents with unconsciousness
c) Known to cause hyperparathyroidism smelling of alcohol. He has multiple bruises in
d) Liver function tests should be monitored various healing stages,right sided weakness with
during treatment. increased reflexes and tonguebite. a) Left sided
e) Safe during breast feeding subdural haemorrhage
19) Which of the following are characteristic b) Left sided cerebral tumour
features of psychosis in mania? c) Right sided cerebral tumour
a) Grandiose delusions d) Right side haemorrhage in pons
b) Mood congruent delusion e) Right sided subdural haemorrhage
c) Delusional perception
MFSU/MSWS 50
26) A 47-year female presented to the clinic e) Continue all drugs and monitor liver
with persistent hypertension. Her last blood enzymes
pressure recorded was 160/90mmHg. She is 30) A 30-year-old male presented with cough, fever
asymptomatic and recently lost her job and pleuritic type chest pain. Examination is
.Investigations revealed Na+ 146, K+- 4.5, Serum unremarkable except left lower lobe consolidation
urea4mmol/L and RBS 7.1mmol/L. What is the which is confirmed by chest x-ray. What is the best
mostappropriate investigation? a) 2D treatment option?
echocardiography a) start IV clarithromycin
b) 24 hours ambulatory blood pressure b) start oral co-amoxiclav
monitoring c) start oral amoxicillin and clarithromycin
c) Transabdominal scan d) Admit to ICU
31) 16-year-old school boy came to the medical
27) 60-year-old male patient with left clinic complainingof recurrent nocturnal cough and
ventricular systoli dysfunction presented with breathlessness on waking. He is currently on DPI
SOB on climbing stairs, butnormal at rest. Beclomethasone 400 micrograms and is using MDI
Commenced on Ramipril and Frusemide.Which Salbutamol 3-4 times per day. He is missing school
drug would improve prognosis? a) Bisoprolol because of his symptoms. On examination, bilateral
b) Digoxin rhonchi were present. Next step in management is, a)
c) Amlodipine Double the dose of beclomethasone
d) GTN b) Change DPI inhaler to MDI inhaler
e) Amiodarone c) Add Montelukast
28) Previously well 60-year-old gentleman d) Add on long acting Theophylline
constricting type chest pain. ECG ST depressions e) Oral steroids for a few days
in anterior leads. Troponin negative and treated 32) Extra intestinal manifestation correlating with
with heparin. What is the best combination of disease activity of IBD? a) Uveitis
drugs for him at discharge? b) Arthritis
a) Aspirin, ACEI, Furosemide, beta blocker, c) Erythema nodosum
Statin d) Nephrolithiasis
b) Aspirin, clopidogrel, ACEI, beta blocker, e) Primary sclerosing cholangitis
Statin 33) 46 years old man with an alcoholic cirrhosis
c) Aspirin, clopidogrel, nitrates, beta presented with progressive abdominal and ankle
blocker, Statin oedema while he is taking spironolactone 100mg and
d) Aspirin, clopidogrel, ACEI, nitrates, K+ frusemide 40mg. On examination, he is icteric, not
blocker pale, pitting ankle oedema up to knee, no flapping
e) Aspirin, clopidogrel, nitrates, Ca+ tremors. WBC 12x10/L, Neutrophil 85%, CRP 24,
blocker, statin WBC on ascitic fluid 450. What is the first
29) 45-year-old male with pulmonary management option? a) Salt free albumin.
tuberculosis started on 4drug regimen. After 4 b) IV ceftriaxone.
weeks of treatment, presents withjaundice and c) Increase dose of diuretics.
abdominal pain, liver enzymes raised morethan 3 d) Large volume paracentesis.
times of upper limit and bilirubin elevated. What e) Low salt diet.
is the best option? a) Discontinue rifampicin and 34) 30 years old female presents with excessive
monitor liver enzymes pruritus and abdominal distension. On examination
b) Discontinue rifampicin and isoniazid and there is free fluids in abdomen and she is deeply
monitor liver enzymes icteric. Her Hb 9g/dl, MCV 104, WBC 3000/mm,
c) Discontinue pyrazinamide and monitor PLT-110x10'/mm3, INR 3, ALT 159.What is the best
liver enzymes next investigation to diagnose hercondition?
d) Discontinue all drugs and monitor liver a) Upper Gi endoscopy.
enzymes b) Serum bilirubin.
MFSU/MSWS 51
c) Serum ferritin. 40) A 35 years old female with recent weight gain,
d) Serum copper. hirsutism,and hypertension. What is the screening
e) Ultrasound abdomen. test to diagnoseto have Cushing's syndrome? a)
Random cortisol level.
35) 48-year-old man with PCKD investigated b) Inferior petrosal vein cannulation.
for headache.WOF is most useful in diagnosing c) Serum ACTH level.
cause for headache? d) Overnight Dexamethasone suppression
a) Radiograph of paranasal sinus. test.
b) Serum creatinine. e) High dose Dexamethasone suppression
c) CT head. test.
d) Lumbar puncture. 41) 20 years old female presented with pallor and
e) MR angiogram. jaundice. Her Hb is 7 g/dl, WBC is 6000/mm,
36) 9-year-old man presented with polyuria PLT- 250x 10/m3
and extreme thirst for 4 weeks. There was no loss a) Blood picture, Reticulocyte count, Serum
of weight, his fasting blood glucose level is 98 bilirubin,Urine hemosiderin.
g/dl. What is the confirmatory a) Serum b) Direct coombs test, Reticulocyte count.
osmolality c) Bilirubin, Urine hemosiderin,
b) Serum sodium Haptoglobin, Osmoticfragility test
c) MRI d) Reticulocyte count, Bilirubin, Blood
d) Water deprivation test picture, Direct
37) 40 years old man presented with SOB for e) Coombs testReticulocyte count, Bilirubin,
1 week to A & E department. Lethargy and LOA Haptoglobin, Hb electrophoresis
for 2 weeks. O/E-pale, B/L ankle oedema, BP - 42) 30-year-old asymptomatic female presenting with
170/100. What are the features suggestive of low platelet count (110), and 3 cm splenomegaly.
CKD rather than AKI? She has no other symptoms. No other abnormalities
a) Increased JVP in examination and full blood count. What is the
b) Hyperkalemia most likely cause?
c) Shrunken kidney on USS a) Thrombotic thrombocytopenic purpura
38) 56 years old male came for routine b) Immune thrombocytopenic purpura
medical check-up. His HbA1c 8.5, Total c) Multiple myeloma
Cholesterol 260, Serum creatinine 1.1.Doctor d) Cirrhosis
advised for dietary control and daily exercise for
3 months. What is the other measure to be 43) 35-year-old female presented with polyarthritis of
prescribed him? small joints of feet and hands or 3 months. Also
a) Dietary control and daily exercise for complains of morning stiffness for 30 minutes. OR -
another month swelling,tenderness present. ESR - 120, RF (+)ve,
b) Repeat HbA1c after 2 months. increased titre ofanti CCP. Most appropriate
c) Review with lipid profile. management? a) Methotrexate + Folic acid weekly
d) Start metformin.
e) Start gliclazide 44) 47 years old retired government officer
39) 38-year-old male, diagnosed with DM for presented.fever for 8 days and confusion for last 2
18 years presented with progressive leg swelling days. At the beginning of the fever, it was associated
and SOB. Which of the following make diabetic with anorexia nausea and lethargy. He is a diagnosed
nephropathy the most likely diagnosis? a) patient with Diabetes mellitus and hypertension and
Proteinuria 3+ on HCT and metformin. He visited to London and
b) Leukonychia Thailand 5 years back. and India 2 months ago. On
c) Creatinine level less than 2 mg/dl examination, he is drowsy, pale, little icteric and has
d) Proliferative retinopathy splenomegaly. Other examinations are normal. What
is the diagnosis?
MFSU/MSWS 52
a) Typhoid. c) Repeat antivenom.
b) Dengue. d) IM adrenaline
c) Viral encephalitis. e) Intubation & ventilation
d) Malaria.
e) HIV seroconversion illness 49) 28-year-old female presented with generalized
rash. It is 12 cm in size, concentric rings surrounded
45) 25 years old patient presented with fever by central lesion.She also complains of pruritus &
for two days. Her Hb 12 g/dl, USS abdomen fluid itching & some of thelesions are seen inside the oral
in the abdomen, Chest X-ray- right pleural cavity. She was recentlytreated for chest infection.
effusion. What is the next step of management? a) What is the condition?
100 ml/hr 0.9% saline infusion. a) Erythema multiforme
b) 500 ml 0.9% saline bolus. b) SLE
c) Dextran 500 ml bolus. c) Secondary syphilis
d) Dextran 500 ml/hr infusion. d) Urticaria
e) Fresh frozen plasma. e) Pemphigus vulgaris
50) In a population of 100 women over 45 years, you
46) A 30-year-old woman with self-ingestion administer a test to detect cancer. Of the 15 people
of poison 2 hoursago. On examination, GCS- having the cancer, the test detected 12. Test also
12/15, BP-90/50 mmHg andconstituted pruritus, detected 3 people who did not have the disease. What
bilateral crepts. What is the best management is the sensitivity and specificity? a) 80,94
option? b) 94,80
a) Gastric lavage. c) 90,90
b) Infusion of 250 ml of 0.9% saline over 20 d) 92, 80
minutes. e) 80, 92
c) Endotracheal intubation. 51) An alcoholic comes to the clinic saying that he
d) Bolus of 3 mg atropine. needs help for abstinence. He is saying that he is
e) Transfer to ICU. considering about changing his drinking behaviour.
He has not taken any stepstowards it. What is the
47) 25 years old male presents with fever & stage of change he is in? a) Action
muscle pain. Hehad been at his friend's birthday b) Contemplation
party till late night. His relatives complain that he c) Pre contemplation
seems to see animals & peoplewho do not exist. d) Decision
BP is 150/80, PR = 126/min. What could be the e) Maintenance
cause? a) Heroin toxicity 52) 55yrs old alcohol abuser admitted for a surgery.
b) Amphetamine toxicity Following day, he became restless, agitated &
c) Codeine toxicity sweating. What is thebest treatment for his acute
d) Tramadol toxicity condition? a) Haloperidol
e) Amitriptyline toxicity b) Chlordiazepoxide
48) A 30 years old farmer admitted following c) Thiamine
snake bite. There is no bleeding manifestations. d) Propranolol
No local implantation, but he is drowsy, GCS- e) Oral lactulose
14/15, BP- 120/80 mmHg, PR- 80
bpm.Antivenom was given successfully and
uncomplicatedly After one hour patient become
drowsier. Respiratory rate is 6/min, BP- 80/60
mmHg. What is the most appropriate next step of
management?
a) Normal saline rapid bolus.
b) Continuous Positive Airway Pressure.
MFSU/MSWS 53
FACULTY OF MEDICINE
UNIVERSITY OF COLOMBO
FINAL MBBS EXAM (MAIN) –2019
2011 A/L BATCH
MEDICINE
c) bronchodilators beta Agonist effective than
1) Features of a large lesion in a dominant antimuscarinic drugs
parietal lobe? d) In acute exacerbation usually pco2 is high &
a) Receptive aphasia B e) In management step 2,add on long acting beta 2
b) Left right orientation. agonist
c) Behavioral abnormalities
d) Contralateral hemiparesis 6) Regarding legionella pneumophila
e) Lower left quadranopia a) Causing diarrhea
b) Lymphocytosis
2) A CSF report shows increased protein c) Treated with ceftriaxone
increased polymorphonuclear leukocytes and d) Causing hyponatremia
possibilities include e) Present in urine
MFSU/MSWS 54
c) adrenal tumors c) Reaction to drugs
d) cushing's syndrome d) Fungal infection
e) thyrotoxicosis e) Lichen planus
11) Features of chronic extra vascular 17) Which of the following is/are examples of
haemolysis mitochondrial inheritance
a) Increased urine hemosiderin a) Charcot marie tooth disease
b) Increased conjugated bilirubin b) Friedreich Ataxia
c) Reticulocytosis d c) Leber hereditary optic neuropathy
d) Hypocellular bone marrow d) Progressive external ophthalmoplegia
e) Pigment stones e) Myoclonic epilepsy with ragged red fibers
(MERRF)
12) Features of sero negative
spondyloarthritis? 18) About cardiac cycle
a) acute anterior uveitis a) ventricular filling is predominantly a passive
b) inflammatory type low back pain process
c) anti CCP b) Blood supply to the heart occurs during systole
d) bilateral symmetrical small joint arthritis c) Tricuspid and mitral valve closes during early
e) Sacroiliitis diastole
d) Pulmonary valve closes before aortic valve during
13) Regarding gout systole
a) Subcutaneous nodules e) Atria contracts during late diastole
b) Symmetrical involvement in small joints
of hands and feet 19) 4 year old male was brought to the emergency
c) Aggravated by alcohol department by an ambulance 1 hour of aphasia a right
d) Metabolic syndrome is a risk factor hemiparesis. On examination blood pressure 178/94,
e) Urate crystals in synovial fluid pulse rate Bpm, irregular rhythm. C scar 1 hour of
arrival showed no early ischemic changes What is the
14) Regarding sepsis most appropriate manager a) Option
a) Is a common cause of distributive shock i) Mechanical thrombectomy
b) If hypotensive, start ionotrope b) Artena rtPA
immediately c) IV labetalol
c) Dobutamine is a choice of inotrope in d) Venous rtPA
sepsis e) low dose aspirin
d) Extremes of age is a risk factor
e) Antibiotics given only after blood culture 20) 45 yrs female, type 2 DM for 2 yrs. Complain of
and ABST result numbness and tingling of feet for 3 months. Also
21) difficulty in walking in darkness and tendency to fall.
15) Regarding complications of leptospirosis: On gliclazide nd metformin.HBA10 6.5 a) critical
a) Pyogenic meningitis illness neuropathy
b) Hemorrhagic pneumonitis b) diabetic neuropathy
c) Myocarditis c) diabetic amyotrophy
d) Pyelonephritis d) folate den
e) Acalculous cholecystitis e) vit B12 deficiency
16) 1Erythroderma can occur in,
a) Eczema 22) 60-year-old female presented with progressive
b) Psoriasis dysphagia & dysarthria for 2 months. On examination
MFSU/MSWS 55
her hand muscles are wasted. Both upper limbs & bilateral fine crepts. What is the a) Allergic
lower limbs are hypertonic & there is no sensory bronchopulmonary aspergillosis.
loss, What is the most likely diagnosis? a) b) Bronchiectasis.
brainstem stroke c) Chronic bronchitis
b) cervical disc prolapse d) Asthma
c) motor neurone disease e) Idiopathic pulmonary fibrosis.
d) multiple sclerosis
e) myotonic dystrophy 27) 40y old male travel guide, presented with progressive
SOB for 3 months. Diagnosed DM for 4 years.On
23) 50 years old male,smoker, developed right metformin and gliclazide is a teetotaler. On
ventricular and inferior STEMI. His PR 60 bpm, examination mild icterus, ankle oedema, shrunken
BP 70/50 mmHg What is the most appropriate next Iver, mild splenomegaly, bilateral fine crepitations.
step to improve BP? a) Dopamine infusion Investigations- hb-10 mcv-84, plt-04, s.Alb-
15,AST-64, alt-84, direct bilirubin 68, alp-110. CXR
b) Dobutamine Infusion
upper lobe diversion, cardiomegaly What is the
c) 0.9% normal saline fluid bolus
diagnosis
d) Noradrenaline infusion
a) Cardiac cirrhosis
e) Temporary pacing
b) Haemochromatosis
c) Hep B
24) 25) 45yrs old presented with sob for
d) NASH
3hrs.pr 140bpm.bp 85/70mmhg.heart sounds
e) Wilsons
normal lungs clear,
a) sat 99% ,ECG rate 138 bpm, narrow QRS
28) 50 year old lady presented with diarrhoea for 6 month
absent p wave. Management?
duration. Also complaining of difficulty in getting up
b) carotid massage
from squatting position. She feels difficulty in
c) IV amiodarone
walking at night. On examination she is pale,
d) DC cardioversion
a) tenderness over the thigh, multiple bruises. What
e) IV adenosine
is the most appropriate investigation to arive at
f) IV verapamil
diagnosis?
b) CECT abdomen
25) 35 years British woman presented to the
c) Endoscopy and biopsy
emergency department. She had 10 hrs flight from
d) FBC
London Colombo 1 week ago. She complains
e) Stool full report.
Dyspnea, cough, chest pain during deep breaths,
f) USS abdomen
She was healthy prior to the trip. Her past medical
history was uneventful apart from hx of two
29) 18 year old girl presenting with acute heart failure.
miscarriage. Most appropriate investigation in
Blood pressure 230/120 mmHg. There are fine
arriving at the diagnosis is? a) D-dimers
a) crepitations in both lungs on auscultation. Hb 8,5
b) CT- pulmonary angiogram g/dL Blood puncture shows normo chromic
c) HRCT chest normo cytic cells. Normal platelets. Urinalysis
d) Echocardiogram E shows Albumin .S. Cr 780 mmol/L.
e) PET-CT chest S. Ca 1.8 What is the most likely diagnosis
b) HUS
26) 55 years old male has presented with c) CKD
progressive dyspnoea, intermittent cough and on d) Renal artery embolism
examination there is clubbing, central cyanosis and e) PSGN
f) IgA nephropathy
MFSU/MSWS 56
b) Continue anticoagulant indefinitely
c) Continue anticoagulant for 3 months and follow
30) A patient with acute kidney injury has up with thrombophilia screening
following investigation results Serum creatinine d) Continue anticoagulant for 2 weeks and convert to
8,2 mg/dl aspirin
a) Sodium 140mmo Potassium 7.9 mmol e) Continue anticoagulant for 3 months.
Serum bicarbonato 16 mmol ECG shows tall T
waves and 35) 30 year old male coming with 6 m hx of lower back
b) broad QRS complexes Which of the pain. Ole limited flexion and extension in
followings are most appropriate next step in the spine.Diagnostic investigation. a) HLAB27
management b) ALP
c) IV calcium gluconate c) X RAY
d) IV furosemide d) SERUM CALCIUM
e) IV insulin and dextrose
f) IV bicarbonate 36) 25 years old lady presents with fever for 4 days with
g) salbutamol nebulization arthralgia and myalgia. On examination BP 120/80
Investigations were as follows WBC 10.5 ( Neutrophil
60%), Hb -.. (low) AST- 140 ,ALT- 135 UFR.
31) MEDICINE SBA a) Red cell + What is the diagnosis?
b) Dengue
c) Leptospirosis
32) A 30-year-old female presented with
d) Hanta
abdominal pain, nausea, dizziness for one-week
e) Typhoid 5
duration and lethargy and weakness for one month
duration. On examination her blood pressure on
37) 23 year old women presented with fever severe
supine position was 110/80 and it was 90/70 on
headache she has rash in the face neck and body,her
seated position. Abdomen was normal with no
blood pressure is 80/50 mmhg she has mild neck
palpable masses and no tenderness. Rest of the
stiffness What is the most likely diagnosis a) DHF
system examination was normal. Her investigation
results were as follows ESR - 8 mmol CRP-10 b) Hantavirus infection
SCP-78 micromoles per liter Blood sugar - 50 c) Meningococcal infection
mg/dl K-5,6 mEq/l Nat. 120 mEq/l What is the d) Rocky mountain spotted fever
most appropriate investigation to confirm the e) Rubella infection
diagnosis
38) 30 year old women presents to emergency unit with a
33) 60 year old man with diabetes onpremixed GCS of 12/15, pinpoint pupils and blood pressure of
insulin (70/30) found unconscious at 4am.what is 70/50. On auscultation she had basal crepitations in
the initial step of management a) 10% dextrose IV both lungs. What might be substance she has taken? a)
b) 50% dextrose IV Clonazepam
c) IM glucagon with 50% dextrose b) Heroin
d) IM glucagon c) Ketamine
e) IV Glucagon d) Lorazepam
34) A 65 year old man was diagnosed to have e) Organophosphate
proximal DVT following a total hip replacement
surgery was started with anticoagulant and PT INR
is maintained at 2.5. What is the most appropriate
next step in the management a) Insert a IVC filter
MFSU/MSWS 57
39) A 24 yr old woman found unconscious in a) Study is most help to say that condition A is in
the bed. RR 8/minOR 80 bpm. Her pupils were deed a risk factor for Condition B b) Case Control
constricted. Best Rx option for her Study
40) a) Atropine c) Control clinical trial
d) Crossover study
b) Flumazenil
e) Cross sectional study
c) Methadone
d) Naloxone
45) 30 years old male presented with a unknown snake
e) Naltrexone
bite. His night leg is swollen and also he is having
41) 52 year old farmer presents with unknown
a) signs and symptoms of acute renal failure. Whole
snake bite while going to the paddy field. He has
blood clotting time is normal. What is the most
progressive sluring of speech and bilateral ptosis.
b) Cobra
There's a fang mark on his left anko. His urine
c) Russell's viper
a) output is satisfactory. Urinalysis showed 12-20
d) Hump nosed viper
on moderate field. What can be the snake? b)
Common krait e) Common krait
c) Cobra f) Saw scaled viper
d) Ceylon krait
e) Hump nosed viper
f) Russell's viper
MFSU/MSWS 58
FACULTY OF MEDICINE
UNIVERSITY OF COLOMBO
FINAL MBBS EXAM (MAIN) – DECEMBER 2019
2012 A/L BATCH
MEDICINE
a. Is a large vessel vasculitis
b. Visual loss is a complication
Multiple choice questions
c. Is treated with high dose steroids
01. Drugs with survival benefit in cardiac
d. Is associated with polymyalgia rheumatica
failure
e. ESR is < 50mm/ 1st hour in majority
a. Digoxin
07. Regarding erysipelas
b. Ivabradine
a. Edges are discrete and merge into the skin
c. Frusemide
b. Caused mainly by group A beta hemolytic
d. Carvedilol
streptococci
e. Spironolactone
c. Affect dermis and superficial subcutaneous
02. Thrombocytopenia occurs in
tissue
a. Chronic liver cell disease
d. Systemic spread can cause multi organ
b. Hemophilia A
dysfunction
c. SLE
e. Penicillins are the treatment of choice
d. Leptospirosis
08. Regarding pneumonia
e. Polycythemia rubra vera
a. ………………………..
03. Renal diseases correctly matched with
b. Erythromycin is an alternative for
the cause
doxycycline in atypical pneumonia
a. Membranous nephropathy – bronchial CA
c. Treatment with ciprofloxacin delays the
b. Acute interstitial nephritis – ibuprofen
diagnosis of tuberculosis
treatment
d. Cough persisting without fever is an
c. Acute tubular necrosis – low dose aspirin
indication to change the antibiotics
d. Acute cortical necrosis – Russel’s viper
e. Chest radiography features resolve before
e. Chronic interstitial nephritis – metformin
the resolution of clinical symptoms
04. In the management of cardiac failure,
09. Prolonged use of inhaled corticosteroids
influencing renin angiotensin aldosterone
cause,
system,
a. Hoarseness of voice
a. Is done by the use of ramipril
b. Fetal growth restriction
b. …………………………
c. Oropharyngeal candidiasis
c. Improves remodeling of heart
d. Diabetes mellitus
d. Should be considered in all patients
e. Sleep disturbances
e. Is indefinitely used in patients with left
10. Hyponatraemia is a feature of,
ventricular failure
Adrenal failure
05. Advantages of LMWH over
Decompensated cirrhosis
unfractionated heparin
Treatment with metformin
No need of monitoring
SIADH
Effect is easily reversible
Thiazide diuretics
Due to its long half life, it doesn't have to be
11. Regarding scrub typhus
given as an infusion
a. Causes meningoencephalitis
Less heparin induced thrombocytopenia
b. Causative organism is Rickettsia
Can be managed as outpatient
prowazekii
06. Regarding giant cell arteritis
c. Presence of eschar is useful in the diagnosis
d. Weekly doxycycline therapy can prevent d. Adenosine deaminase will increase in TB
the disease peritonitis
e. Causes thrombocytopenia e. Portal hypertension has a SAAG score more
12. Regarding tuberculosis than 1.1
a. People vaccinated with BCG rarely get TB 18. Regarding gastric mucosa
b. Isolation of bacteria in sputum is essential a. Secreted by G cells of the stomach
for diagnosis b. Acetylcholine stimulates gastrin secretion
c. ……………………………. c. Ranitidine blocks the action of H+ / K+
d. If spine is involved, duration of therapy 9 ATPase
months d. Helicobacter pylori stimulates gastrin
e. It's a notifiable disease in Sri Lanka secretion
13. Postural hypotension is a feature of, e. Gastric acid secretion is inhibited by
a. Addison’s disease prostaglandin E2
b. Prazosin therapy 19. Regarding hepatitis virus serology
c. Diuretic therapy a. A positive Hepatitis B Surface antigen
d. Pheochromocytoma indicates active infection
e. Steroid therapy b. IgM antibodies to hepatitis B core antigen
14. Regarding a lesion in the posterior is positive in acute infection
column c. Hepatitis B surface antigen and anti HBs
a. ………………………….. antibodies can co-exist
b. It causes ipsilateral joint position sensory d. Presence of Hepatitis B e antigen is
loss indicative of infectivity
c. Causes contralateral hemiparesis e. Anti Hbs antibody in blood shows presence
d. Tendon reflexes are absent of immunity
e. Rhomberg test is positive 20. Regarding acute kidney injury
15. Which of the following are true Adequate replacement of the volume will
regarding X linked dominant diseases? reverse pre renal failure Brown muddy
When females are affected, they show more granular casts- glomerular nephritis???
signs. Renal biopsy is contraindicated in anuria
Male to male transmission can occur Pericarditis is a complication
Consanguinity does not affect the outcome Diuretics improve survival
Females transmit only to males
Affected females will transmit the disease to 25
% of their off springs. Single best responses
16. Regarding Guillen Barre Syndrome, 01. A 35-year-old female with low grade
a. Presence of a sensory level is a fever and headache for 2 weeks. Imaging
characteristic feature shows enlarged ventricles and basal
b. Exaggerated tendon reflexes present exudate. What would you expect in CSF
c. Cerebrospinal fluid protein is increased report?
d. MRI scan is essential in diagnosis a. Lymphocytic predominant, low glucose,
e. Steroids are the mainstay of treatment high protein
17. Regarding ascites b. Lymphocytic predominant, normal glucose,
a. More than 500ml causes shifting dullness low protein
b. More than 100ml can be seen in USS c. Lymphocytic predominant, normal glucose,
c. Polymorphs 250/µl is diagnostic of normal protein
bacterial peritonitis d. Neutrophil predominant, normal glucose,
normal protein
MFSU/MSWS 60
e. Neutrophil predominant, low glucose, high b. Chronic obstructive pulmonary disease
protein c. Pulmonary tuberculosis
02. A 35-year-old male, strong family history d. Allergic Broncho-pulmonary aspergillosis
of diabetes presented for a medical e. Bronchiectasis
checkup. BP – 130/90, CVS and RS 05. 56y old male. Known diabetic. Past
examination normal. history of rheumatic heart disease.
Investigation results, Presented with sudden onset LL weakness.
FBS – 140 O/E bilateral flaccid paralysis. Power 2/5.
HbA1c – 7.2% Pain sensation impaired up to umbilicus
Total cholesterol - 220 (>200) level. Intact joint position.
LDL cholesterol – 160 (>150) What is the most probable diagnosis?
AST – 60 (10-40) a. GBS
ALT – 80 (10-60) b. Transverse myelitis
Serum creatinine – normal c. Cauda equina syndrome
a. Start metformin d. Anterior spinal artery occlusion
b. Start sitagliptin e. Diabetic amyotrophy
c. Start gliclazide 06. In Cushing syndrome, what feature is
d. Start Insulin more suggestive of an ectopic ACTH
e. Review with fasting blood sugar after one secreting tumor? a. Hyponatremia
month without starting medication b. Excessive cortisol
03. A 36-year-old male, with a history of an c. Glucose intolerance
unprotected sexual act 4 weeks ago, d. Metabolic alkalosis
presents to STD clinic with a painless ulcer e. Refractory hypokalemia
on penis. Previously well. What’s the 07. A 22-year-old male, returned from India
Possible lesion? a. Chancroid 1 month back. Now has intermittent fever
b. Genital herpes with chills and rigors, headache, arthralgia
c. Gonorrhea and myalgia for 1 week. On examination
d. Syphilis mild hepatomegaly with no lymph node
e. Chlamydia enlargement. FBC: WBC normal, Hb –
04. A 35-year-old male presents with 10.8, platelets – 120,000.
wheezing and productive cough of 4 days Most appropriate initial investigation to
duration which is not responding to inhaled come to a diagnosis?
salbutamol and high dose beclomethasone. a. Urine microscopy
He used to smoke 5 cigarettes per day since b. HIV antibody
25 years of age and stopped 1 year back as c. MAT for leptospirosis
advised by a doctor. O/ E dyspneic, diffuse d. Blood culture
BL rhonchi, normal CVS examination. e. Malaria blood film
CXR normal. 08. A 50-year-old male from UK visiting Sri
ESR 25 Lanka for a short vacation presented with
WBC 15 acute onset difficulty of breathing, cough
N 50% and left sided chest pain. Well controlled
L 38% asthma on combined inhalers, smoking 10
E 12% pack years. On examination, no fever, Pulse
HRCT - Right sided dilated primary and rate was 120 and Blood pressure 112/72
secondary bronchi What is the most mmHg, Respiratory examination was
probable diagnosis? unremarkable. SpO2 was 88%. ECG -
a. Asthma
MFSU/MSWS 61
Sinus tachycardia with V1-V4 leads T b. 95% chance that the sample mean will fall
inversions. Full Blood Count normal. within the confidence interval
Most probable diagnosis? c. ………………………
a. Acute exacerbation of asthma d. 95% chance that the population mean will
b. Acute exacerbation of COPD fall within the confidence interval
c. Myocarditis e. 0.05 probability that the population mean
d. Acute coronary syndrome will fall within the confidence interval
e. Pulmonary embolism 13. 25 year old male returned from holiday
09. A 65-year-old man presented with acute came with a left sided chest pain for 1 day
onset right sided upper limb and lower limb duration. He had flu for 3 days. The pain
weakness of 2 hours duration and he had was episodic and ceases on expiration. ECG
difficulty in understanding what other was done. There were ST elevations in
people are speaking. Blood pressure was anterior and lateral leads. CRP=30,
170/110mmHg and pulse rate 88 bpm. WBC=11, troponin +. What is the most
What’s the next step of management? likely diagnosis?
a. …………………… a. Acute coronary syndrome
b. Infusion of IV labetalol b. Acute pericarditis
c. IV tissue plasminogen activator c. Pneumothorax
d. Start him on warfarin 5mg/day treatment d. Pleural effusion
e. Treat with S/C enoxaparin e. ………………..
10. A 35-year-old man with chronic bloody 14. 35-year-old female presents with icterus
diarrhea. Which of the following favor the and generalized pruritus. Her INR is 2.5,
diagnosis of Crohn’s disease over ulcerative total bilirubin 2.5 and ALP 800. USS is
colitis? suggestive of CLCD. Best investigation to
a. Pancolitis check for aetiology? a. anti-mitochondrial
b. Perianal abscess antibody
c. Past history of perianal fistula b. Hepatitis B surface antibody
d. Elevated fecal calprotectin c. Hepatitis C antibody
e. Positive family history d. Serum ceruloplasmin level
11. A 40-year-old male presented with e. Anti – smooth muscle antibody
periorbital edema for 2 days. O/E : BP – 15. 45-year-old male presenting with first
160/100, RS examination – bilateral basal episode of UTI. Culture positive for E. coli.
crepitations What is the next best investigation?
UFR findings – a. Cystoscopy
Protein + b. DMSA
RBC ++ c. Screening for STD
Granular casts d. USS KUB
Pus cells – nil Serum creatinine – 1.8 e. IV urogram
Most probable diagnosis? 16. 20-year-old previously healthy female,
a. Acute interstitial nephritis presenting to hospital after ingesting an
b. Acute glomerulonephritis unknown number of paracetamol tablets.
c. …………………………. What is the next best management option?
d. Nephrotic syndrome a. Gastric lavage followed by NAC
e. Undiagnosed CKD b. N-acetyl cysteine
12. Which of the following best describes the c. Active charcoal
confidence interval? d. Methionine
a. Ranges of the means of the population
MFSU/MSWS 62
e. Wait for one more hour and do according to 20. A patient presenting 4 hours after
nomogram ingestion of an organophosphate. BP –
17. 45-year-old IV drug user presenting with 80/50, PR – 56, RR – 23, SpO2 – 88%. On
intermittent fever and a murmur on left auscultation crackles were heard. Next
lower sternal edge. Diagnosed to have most appropriate step of management?
infective endocarditis. Most probable a. 250ml 0.9% NaCl
causative organism? a. Streptococcus b. Nebulize with salbutamol
epidermidis c. Oxygen via venturi mask
b. Streptococcus bovis d. Atropine 3mg IV bolus
c. Streptococcus viridans e. Pralidoxime 1g IV over 30 minutes
d. Pseudomonas aeruginosa 21. 50-year-old patient presenting with
e. Staphylococcus aureus central chest pain which is relieved on
18. 15-year-old girl presenting with past expiration. CVS and RS examination
history of intermittent jaundice for last few normal. ECG – prominent R wave and ST
years. depression in V1 and V2. Most probable
Investigations are as follows. diagnosis?
Hb – 13.2 a. Brugada syndrome
WBC – 6.3, N – 62%, L – 34% b. NSTEMI
Platelets – 282 c. Posterolateral MI
LDH – 200 d. True posterior MI
Direct bilirubin – 6 e. Myocarditis
Indirect bilirubin – 34 22. 40-year-old banker who has taken ‘’long
AST – 14 term over the counter’’ medication for a
ALT – 26 chronic headache presented to the ward
INR – 1.2 with headache. BP – 120/80. His father has
ALP – 140 died due to a stroke at the age of 40 years.
GGT – 25 On examination, he has small ecchymotic
Most likely diagnosis? patches over the tongue and lateral aspects
a. Gilbert syndrome of fingers. Most likely condition?
b. Biliary atresia a. PCKD
c. Dubin Johnson syndrome b. Chronic drug abuse causing gastric erosion
d. Cholecystitis c. Chronic drug abuse causing chronic kidney
e. Rotor syndrome disease
19. 18-year-old school boy present with one d. Drug induced aplastic anemia
day history of reduced urine output. He is e. Hereditary hemorrhagic telangiectasia
previously healthy and is a long-distance 23. 45-year-old female complains of low
runner. grade fever, early morning joint stiffness
BU – 10 and pain for more than one hour duration.
Serum creatinine – 900 She also complains of pain and stiffness in
Na – 132 neck, hip joint and pelvic girdle. Most likely
K–6 diagnosis?
Possible etiology for his renal failure? a. Infective endocarditis
a. CKD b. Polymyalgia rheumatica
b. Dehydration leading to hypotension c. Rheumatoid arthritis
c. Interstitial nephritis d. SLE
d. Rhabdomyolysis e. Tuberculous polyarthritis
e. ……………………………
MFSU/MSWS 63
24. 18-year-old man presented following a 29. 50 kg female presents with fever, myalgia
hump nosed viper bite. No systemic and retro orbital pain, tenderness over the
envenomation features noted. Only two right hypochondrium and no urine output
fang marks are present. What is the next for the past few hours duration. On
step in management? a. Arrange WBCT examination BP – 80/60, PR – 96, fine
b. Arrange renal function tests crepitations in lung bases. Next step of
c. Start antivenom serum management?
d. Monitor for 24 hours a. 250ml of normal saline bolus
e. Hemodialysis b. 250ml of Hartman solution bolus
25. 32-year-old female presents with swollen c. 250ml blood transfusion
lips for 5 days. She is on oral hypoglycemic d. 500ml dextran bolus
medication for diabetes which is well e. 500ml normal saline bolus
controlled. No other medical co morbidities. 30. 45-year-old male presented with low
No pruritus or past allergic history. She has grade fever for 5 days associated with
recently used a new hair dye one week back. headache and myalgia. There has been a
What is the most likely diagnosis? dry cough for the past few days. On
a. Urticaria examination he is mildly icteric and has
b. Angioedema bilateral crepitations. Chest x ray reveals
c. Anaphylaxis bilateral diffuse opacities over the lower
d. Erythroderma lobes of lungs. Most possible causative
e. Pemphigus organism?
26. …………………….. a. Streptococcus pneumoniae
27. 40-year-old male with history of back b. Streptococcus epidermidis
pain currently presents with a painful knee c. Legionella pneumophila
joint that has persisting morning stiffness d. Mycoplasma pneumoniae
for more than two hours duration. Which of e. Staphylococcus aureus
the following describes the most likely
condition?
a. Ankylosing spondylitis
b. Psoriatic arthritis
c. ………………
d. Reiter’s syndrome
e. Rheumatoid arthritis
28. 24-year-old female bank officer has a
weight gain of 8kg over 3 months. She also
complained of lethargy and
oligomenorrhea. BP – 170/100. K+ - 3.3, Na+
- 146, TSH - normal, Hb – normal.
What is the most likely cause for her elevated
blood pressure?
a. Conn syndrome
b. Cushing syndrome
c. Hypothyroidism
d. Metabolic syndrome
e. Obstructive sleep apnoea
MFSU/MSWS 64